SlideShare a Scribd company logo
1 of 19
Situation 1: Information dissemination is an integral element of health promotion, and disease prevention. The nurse 
is in the best position to do health education activities. 
1. A nurse is providing instructions to a pregnant client with genital herpes about measures that need to be 
implemented to protect the fetus. The nurse tells the nurse that: 
a. Daily administration of acyclovir (Zovirax) is necessary during the entire pregnancy 
b. Total abstinence from sexual intercourse is necessary during the entire pregnancy 
c. Sitz baths need to be taken every 4 hours while awake if vaginal lesions are present 
d. A cesarean section will be necessary if vaginal lesions are present at the time of labor 
2. While the nurse is counseling a group of mothers about sexually transmitted diseases, one mother asks the nurse 
which sexually transmitted disease is most detrimental to their baby's safety during pregnancy? The nurse should 
respond that it would be: 
a. Gonorrhea c. Genital herpes 
b. Syphilis d. Trichomonas vaginalis 
3. During an infection control seminar, the speaker specified that prevention of the spread of HIV include the 
following measures EXCEPT: 
a. Patients with AIDS should be isolated 
b. Blood and other specimens should be labeled AIDS Precaution 
c. Needles should be disposed into a puncture resistant container 
d. Blood spills should be cleaned with chlorox 
4. A nurse is giving health education to a client diagnosed with HIV. The nurse determines that the client does not 
need further teaching if the client states that the most effective method known to control the spread of HIV infection 
is: 
a. Premarital serologic screening 
b. Prophylactic treatment of exposed people 
c. Laboratory screening of pregnant women 
d. Sex education about preventive behaviors. 
5. The following are preventions of gonorrhea transmission EXCEPT: 
a. Sex education 
b. Case finding 
c. Incidence to be reported to health authorities 
d. Administration of ophthalmic prophylaxis, as ordered 
Situation 2: The nursing process is applied in any health care setting. 
6. A child is diagnosed with scarlet fever. The nurse assesses the child knowing that which of the following is not a 
clinical manifestation associated with this disease? 
a. Pastia's sign 
b. White strawberry tongue 
c. Edematous and beefy, red colored pharynx 
d. Koplik spots 
7. A nurse provides instructions to the mother of a child with mumps regarding respiratory precautions. The mother 
asks the nurse the length of time required for the respiratory precautions. The nurse most appropriately responds 
that: 
a. “Respiratory precautions are necessary for the entire time of illness.” 
b. “Respiratory precautions are necessary until the swelling is gone.” 
c. “Respiratory precautions are indicated during the period of communicability.” 
d. “Respiratory precautions are indicated for 18 days after the onset of parotid swelling.” 
8. A 6-month-old infant receives DPT immunization at the well-baby clinic. The mother returns home and calls the 
clinic to report that the infant has developed swelling and redness at the site of injection. The nurse tells the mother 
to: 
a. Apply a warm pack to the injection site 
b. Bring the infant back to the clinic 
c. Apply an ice pack to the injection site 
d. Monitor the infant for fever 
9. A nurse is preparing the plan of care for a patient with herpes genitalis. What would be the priority nursing 
diagnosis for the patient?
a. Disturbed Sleep pattern 
b. Imbalance in Nutrition: Less than Body Requirements 
c. Alteration in Comfort: Pain 
d. Ineffective Breathing Pattern 
10. The nurse on duty, based on her assessment findings suspects the presence of diphtheria. Which of the following 
will confirm her suspicion? 
a. Pharyngotonsilar congestion 
b. Grayish membrane at hard palate 
c. Whitish plaque at buccal mucosa 
d. Increased lacrimation 
Situation 3: There are 170 Bilhariasis endemic municipalities. You are assigned in one of these municipalities. 
11. In order to confirm the diagnosis of Snail fever, you advise patient to have, which of these examination: 
a. X-ray of the abdomen 
b. Urinalysis 
c. Stool examination 
d. CBC 
12. You know that the mode of transmission of Katayama fever is: 
a. Contact with affected stray animals 
b. Contact with water infected with cercaria 
c. Infected flies and rodent 
d. Use of sanitary toilets 
13. Which among the following is the drug of choice for Bilhariasis? 
a. Biltricide c. Chloramphenicol 
b. Hetrazan d. Tetracycline 
14. The following are preventive measures for Schistosomiasis EXCEPT: 
a. Use of safe water 
b. Avoid bathing and washing in infested waters 
c. Use of sanitary toilets 
d. Elimination of breeding sites of mosquitoes 
15. Which of the following is NOT a complication of Bilhariasis? 
a. Liver cirrhosis and portal hypertension 
b. Cor pulmonale, pulmonary hypertension 
c. Meningitis and hepatomegally 
d. Ascitis and renal failure 
Situation 4: Endemic malaria occurs in the topic and subtopic areas where socioeconomic condition is very poor. 
16. During your assessment, which among the following signs and symptoms need referral to a secondary or tertiary 
facility? 
a. Sweating and headache 
b. Icterus and shock 
c. Fever and chills 
d. Renal or liver failure 
17. Laboratory confirmation of malaria is done on a blood film. What do you expect to see in the film? 
a. Antibodies c. Malarial purinates 
b. Malarial parasites d. Antigen 
18. The mode of transmission of this disease is through the bite of an infected female mosquito called: 
a. Aminophylline c. Anopheles 
b. Aedes poecillus d. Aedes egypti 
19. Which of the following is NOT an anti-malarial drug? 
a. Sulfadoxine c. Tetracycline 
b. Amoxicillin d. Quinidine
20. As a preventive measure for malarais, you educate people living in malaria endemic areas to do which of the 
following? 
A. Avoid going out between 12nn to 3am 
B. Take chloroquine tablets once a day 
C. Apply insect repellant on house walls 
D. Use long sleeved shirts when going out at night 
E. Plant neem tree in their backyards 
F. Clear hanging branches nears rivers 
a. A, B, C, D, E, F c. B, C, D, E, F 
b. C, D, E, F d. B, C, D, E 
Situation 5: A nurse is having her duty in a public health clinic. She encounters multiple cases of sexually transmitted 
diseases. 
21. A nurse is collecting data from a prenatal client. The nurse determines that which of the following places the 
client into the high risk category for contracting human immunodeficiency virus? 
a. Living in an area where HIV infections are minimal 
b. A history of IV drug use in the past year 
c. A history of one sexual partner within the past 10 years 
d. A spouse who is heterosexual and had only 1 sexual partner in the past 10 years 
22. Which among the following informations is true of HIV? 
a. It can be transmitted via body fluids such as blood, semen, urine, and perspiration. 
b. Blood, semen, and breast milk have higher concentrations of HIV than urine, saliva, vomitus and stool 
c. A client who by history may be exposed to HIV but test negative for HIV antibodies can no longer infect others 
d. Enzyme-linked immunosorbent assay (ELISA) is the confirmatory test for HIV 
23. Hepatitis B infection is established by the presence of hepatitis B antigen-antibody systems in the blood. Which of 
the following is NOT true? 
a. Presence of HBsAG is the serological marker to establish the diagnosis of hepatitis B. 
b. If the serological marker is present in the blood after 3 months, it indicates a carrier state or chronic hepatitis. 
c. The presence of anti-HBS indicates recovery and immunity to hepatitis B. 
d. Presence of HbeAG determines the infective state of the client 
24. The nurse is talking to a young female client in the health clinic who is concerned she may have sexually 
transmitted disease. The nurse commends her for seeking medical care. The nurse explains that the major reason 
treatment of majority of STDs is delayed because: 
a. Client is embarrassed 
b. Symptoms are though to be caused by something else 
c. Symptoms are ignored 
d. The client does not manifest signs and symptoms 
25. The nurse is very active in contact tracing and epidemiologic treatment of all gonococcal identified contacts to 
prevent: 
a. Development of resistant strains 
b. Asymptomatic infections 
c. Non-venereal transmission 
d. Reinfection 
Situation 6: Avian Influenza is an infectious disease of birds that can cause serious illness in humans. 
26. The importation of chicken from countries with outbreak of avian flu is banned by which law? 
a. RA 280 c. PD 280 
b. EO 280 d. AO 280 
27. The major causes of death in avian flu are the following except: 
a. Severe viral pneumonia 
b. Respiratory distress 
c. Multiple organ failure 
d. Dehydration from severe vomiting and diarrhea
28. A suspected case of avian flu would be identified if a person exhibits which of the following manifestations? 
a. Body weakness, fever, vomiting, diarrhea, cough, and anorexia 
b. Fever, body weakness cough, dyspnea, and sore throat 
c. Fever, cough, sore throat, diarrhea, bloody stool, and hematemesis 
d. Hemoptysis, difficulty breathing, sore eyes, vomiting and diarrhea 
29. It is necessary to institute which of the following measures to birds suspected of being exposed to the virus? 
a. Vaccination of poultry 
b. Killing of all infected or exposed birds 
c. Mixing antiviral to feeds and water supplied to poultry suspected of being exposed to avian flu 
d. All of the above 
30. Incubation period for avian influenza is: 
a. 10 days c. 21 days 
b. 3 days d. 7 days 
answers------------------------------------------------------------- 
1. D. In mothers with genital herpes, the fetus can be contaminated after membranes rupture or with vaginal 
delivery. Therefore, for women with active lesions, either recurrent or primary at the time of labor, delivery should be 
by cesarean to prevent the fetus from coming into contact with the genital herpes. 
A. The safety of acyclovir had not been established during pregnancy and should only be used when life-threatening 
infection is present. 
C. Abstinence should be advised while the lesions are present, until they become culture- negative. 
D. This promotes healing. 
2. A. Syphilis may cross the placenta, causing congenital syphilis. It usually leads to spontaneous abortion, and it 
increases the incidence of mental subnormality and physical deformities in fetus. 
B. In gonorrhea, the fetus is contaminated at the time of delivery. It places the neonate at risk for ophthalmia 
neonatorum, pneumonia and sepsis, but these can be avoided via cesarean delivery. 
C. In genital herpes, fetus is contaminated after membranes rupture or with vaginal delivery. Mother should deliver 
via cesarean section. 
3. A. Patients with AIDS are immunocompromised. Reverse isolation is required. But this does no prevent the spread 
of AIDS. 
B, C and D are universal preventive measures for AIDS. 
4. D 
5. D. This does not prevent transmission of gonorrhea. It prevents the occurrence of ophthalmia neonatorum. 
6. D. Koplik spots are associated with rubeola. 
A. Pastia's sign describes a rash that is seen in scarlet fever that will blanch with pressure except in areas of deep 
creases and folds of the joints. 
B The tongue is initially coated with a white furry covering with red projecting papillae (white strawberry tongue). 
C. The pharynx is edematous and beefy red in color.
7. A. Mumps is transmitted via direct contact or droplet spread from an infected person and possibly by contact with 
the urine. Respiratory precautions are indicated during the period of communicability. 
8. C. Occasionally, tenderness, redness, or swelling may occur at the site of the injection. This can be relieved with ice 
packs for the first 24 hours followed by warm compresses if the inflammation persists. 
9. C. Genital herpes is characterized by painful vesicles affecting the external genitalia, vagina and cervix. 
10. B. The presence of a grayish membrane on the hard palate, or pseudomembrane is a pathognomonic sign of 
diphtheria. Whitish plaque at buccal mucosa, or koplik spots, are seen in a rubeola infection (measles). 
11. C. The male and female parasites live in blood vessels of intestines and liver, but the eggs are laid in the terminal 
capillary vessels in the submucosa of the intestines, and through the ulcerations reach the lumen of the intestines and 
pass out with the feces. 
12. B. Infection occurs when skin comes in contact with contaminated fresh water where cercariae (free swimming 
larval forms of schistosomes) are living. 
13. A. Praziquantel (Biltricide) is the drug of choice for Schistosomiasis. 
B. Hetrazan is the drug of choice for Filiariasis 
C. Chloramphenicol is the drug of choice for Typhoid fever 
D. Tetracycline is the drug of choice for Cholera 
14. D. This is a preventive measure for vector-borne diseases. Schistosomiasis is a water-borne disease. 
A. Drinking water should be free of cercariae. 
B. Schistosomiasis can be transmitted through exposure to contaminated water. 
C. The use of sanitary toilets prevents viable eggs from reaching bodies of water containing intermediate snail host. 
15. C. Meningitis is not a complication of Bilhariasis. 
16. B. Malarial parasites invade and destroy red blood cells. 
17. B. As sporozoites from the salivary gland of a female Anopheles mosquito are injected under the skin, they travel 
into the bloodstream to the liver and mature within hepatocytes. Up to 30,000 parasites are then released into the 
bloodstream as merozoites and produce a symptomatic infection as they invade and destroy red blood cells.
18. C. 
B. This is the vector for Filariasis. 
D. This is the vector for Dengue. 
19. B. 
A. Sulfadoxine (Sulfalene) 50 mg 
C. Tetracycline hydrochloride 250mg/capsule 
D. Quinidine sulfate 300 mg table; 
Quinidine hydrochloride 300 mg/mL, 2 ML ampule 
20. B. A and b are not included. The vector's peak biting hours are 9pm to 3am. Chloroquine is taken at weekly 
intervals. 
21. B. Women who fall into the high-risk category for HIV infection include those with persistent and recurrent 
sexually transmitted diseases or a history of multiple sexual partners, and those who use or have used IV drugs. 
A, C and D do not contribute to the incidence of contracting HIV. 
22. B. This is true of HIV. 
A. Perspiration does not contain the virus. 
C. This is not always true for HIV. Clients may be infected with the virus, but has not yet produced antibodies, thereby 
testing negative, but being capable of infecting others. It usually takes 6 to 12 weeks (other books: 1 to 3 months) for 
a host to manufacture detectable HIV antibodies. 
D. Western blot is the confirmatory test for HIV. 
23. B. Presence of hepatitis B surface antigens (HBsAG) in the blood after 6 months indicates a carrier state or chronic 
hepatitis. 
A, C and D are true.Hepatitis B early antigen is detected in the blood about 1 week after the appearance of HBsAG. Its 
presence determines the infective state of the clien 
24. A 
25. B. 80% of gonoccocal infection particularly in females have no symptoms. 
26. B 
27. D. Avian flu is not associated with vomiting and diarrhea.
28. B 
A and D. Vomiting and diarrhea are not associated with bird flu. 
C. Diarrhea, bloody stool, and hematemesis not associated with bird flu. 
29. B. One control measure for Avian flu is the rapid destruction (culling or stamping out) of all infected or exposed 
birds. Other control measures include proper disposal of carcasses; quarantining; rigorous disinfection of farms; and 
restriction on the movement of live poultry. 
30. B. The incubation period ranges 2-4 days. 
Questions-------------------------------------- 
Situation 13. Leprosy is an ancient disease and is a leading cause of permanent physical disability among the 
communicable diseases. It is a chronic mildly communicable disease that mainly affects the skin, the peripheral 
nerves, the eyes and mucosa of the upper respiratory tract. 
61. Assessment is done upon seeing a patient with leprosy. The following are early signs and symptoms, except: 
1. Appearance of a skin patch 
2. Loss of sensation on skin lesion 
3. Inability to close eyelids 
4. Hair growth over the lesion 
5. Painful nerves 
6. Loss of eyebrow 
7. Ulcers that do not heal 
8. Sinking of the nose bridge 
a. 3, 5 and 7 c. 3, 6 and 8 
b. 2, 5 and 8 d. 2, 6 and 7 
62. A person can get leprosy through the following except: 
1. Inhalation of the infectious organism through a broken skin 
2. Repeated skin to skin contact with untreated individual 
3. Inhalation of the infectious organism from the nasal discharges of an untreated individual 
4. Contact with articles soiled with discharges from lesions of leprosy patient 
a. 1 only c. 4 only 
b. 1 and 3 d. 3 and 4 
63. Philip is a 55 year-old resident who is affected by leprosy and is on schedule for Multidrug Therapy. Which daily 
treatment is usually prescribed by a physician for lepromatous and borderline type in 28 days? 
a. Dapsone 10 mg and Rifampicin 600 mg 
b. Clofazimine 50 mg and Rifampicin 50 mg 
c. Rifampicin 150 mg and Dapsone 50 mg 
d. Clofazimine 50 mg and Dapsone 100 mg 
64. Only in rare instances is there really a need to use laboratory and other investigations to confirm a diagnosis. 
However, when a clinical diagnosis is doubtful. This optional procedure is done to prevent misclassification and wrong 
treatment. 
a. Direct Sputum Smear Microscopy (DSSM) 
b. Slit Skin Smear (SSS) examination 
c. Mass Blood Smear (MBS) collection 
d. Purified Purine Derivative (PPD) test 
65. Preventive aspect of care is a priority consideration. Which of the following vaccine is also preventive measure? 
a. BCG c. Gammaglobulin 
b. DPT d. ATS 
Situation 14. Nurse Lorie is taking care of Anne, 15 months old, who is admitted to the hospital with Rubeola. She has 
conjunctivitis, coryza, cough, rashes and temperature of 38ºC.
66. Which of the following statements is true of Rubeola? 
1. It is caused by a filterable virus 
2. The prodrome period lasts for about 2 weeks 
3. The rashes doesn't end in a branny desquamation 
4. It is an acute highly communicable infection 
5. The rash usually begins behind the ears and neck 
6. The cause of death is usually from complications 
7. Serum antibodies appear on the third day of the incubation period 
8. A morbilliform rash appears on the 3rd or 4th day 
a. 1, 2, 3, 5 and 6 c. 2, 3, 4, 5 and 7 
b. 3, 4, 5, 7 and 8 d. 1, 4, 5, 6 and 8 
67. Nurse Lorie knows that the incubation period of Rubeola is: 
a. 10 days from exposure to appearance of fever and about 14 days until rash appears 
b. 2-3 weeks, commonly 13 to 17 days 
c. 12 to 26 days, usually 18 days 
d. Varies from 3 days to 1 month or more, falling between 7-14days in high proportion of cases 
68. Anne's immunity from her mother lasted only 6 months. She was brought by her mother because of spots in her 
buccal mucosa. What do you call these spots? 
a. Pseudomembrane c. Koplik spots 
b. Rose spots d. Forscheimer's spots 
69. The characteristic signs of measles are which of the following group? 
a. Skin eruptions which are abundant on covered areas of the body than on the exposed areas 
b. Rashes which spread from the face to the trunk and limbs, conjunctivitis, high fever and tiny white spots in the 
mucosa inside the cheek 
c. Vomiting, headache, fine petechial and morbilliform rashes and epistaxis 
d. Severe backache and rashes which are more abundant on extremities than on the trunk 
70. A nursing intervention that is important in caring for Anne would include the following, except: 
a. Administer as prescribed antipyretics to reduce fever 
b. Ensuring that the room is well lighted 
c. Wearing face mask when administering nursing care to the child 
d. Increase fluid intake of the child by drinking water frequently 
Situation 15. Vangie is a 15 year-old child who is spending Christmas Vacation in Pangasinan. A week before the 
vacation ends, she developed fever, diarrhea and rose spots on her abdomen. 
71. Based on Vangie's clinical findings, Nurse Ulysses will suspect Vangie to have contacted which infection? 
a. Dysentery c. Cholera 
b. Amoebiasis d. Typhoid fever 
72. Vangie's condition is caused by a: 
a. Virus c. Helminth 
b. Bacteria d. Protozoa 
73. Nurse Ulysses provided the family with health education to prevent transmission of infection as the disease can 
be transmitted to family members through the following, except: 
a. Flies 
b. Contaminated food and water 
c. Airborne transmission 
d. Direct contact 
74. The characteristic signs of the said condition in #71 are which of the following? 
1. Intermittent fever 
2. Continued fever 
3. Loss of appetite 
4. Slow pulse 
5. Rose spots on trunk and abdomen 
6. Diarrhea 
7. Rice watery stool 
8. Bloody stool
a. 2, 3, 4, 5 and 6 c. 1, 3, 4, 5 and 7 
b. 1, 3, 4, 5 and 6 d. 2, 3, 4, 5 and 8 
75. As a preventive control measure for the said condition in #71, which of the following must be discussed in the 
health education program for the pubic? 
1. Sanitary disposal of human feces and maintenance of fly proof latrine 
2. Proper food handling and preparation 
3. Removal of stagnant water in empty water drums and flower pots 
4. Use of repellants and insecticides 
a. 1 and 4 c. 3 and 4 
b. 2 and 3 d. 1 and 2 
Situation 16. According to a nationwide survey on soil-transmitted helminthiasis (STH) among children aged 12-71 
months old done by the DOH, UP and UNICEF in 2003, it showed a cumulative prevalence rate of 66 percent. Studies 
have shown that STH infected children are prone to acquire other infections. There is a need to sustain mass 
deworming for 3 consecutive years to immediately control intestinal parasitism and other parasites among these 
infected children at the critical time of their growth and development. 
76. Antihelminthic drugs used include: 
a. Albendazole and Mebendazole 
b. Metronidazole and Albendazole 
c. Cotrimoxazole and Metronidazole 
d. All of the above 
77. It is a parasitic disease that has similar manifestation with Pulmonary Tuberculosis. What is it? 
a. Trichuriasis c. Ascariasis 
b. Filariasis d. Paragonimiasis 
78. The treatment of choice for the condition in #77 is: 
a. Bithionol c. Praziquantel 
b. Ethambutol d. Metronidazole 
79. The three major causes of intestinal parasitic infections in the Philippines are the following, except: 
a. Trichuriasis c. Ancylostomiasis 
b. Taeniasis d. Ascariasis 
80. This parasite enters the intestinal mucosa and could be spread through autoinfection. This is typically exemplify 
by: 
a. hookworm c. whipworm 
b. tapeworm d. pinworm 
Situation 17. Lord Voldemort complained of dizziness, vomiting, headache and numbness of the face especially 
around the mouth after 45 minutes of eating shellfish. He was rushed to the hospital and was diagnosed with 
Paralytic Shellfish Poisoning (PCP) or Red Tide Poisoning. 
81. In addition to the above mentioned signs and symptoms manifested by Lord Voldemort, he may also manifest the 
following: 
a. Blurred vision and rapid rise in the blood pressure 
b. Seizures, bradycardia, hematemesis and chest pain 
c. Tachycardia, dysphagia, paresthesia and ataxia 
d. Floating sensation, hypertension, bradycardia and abdominal pain 
82. Which of the following statement is not true about red tide poisoning? 
a. The toxin is totally destroyed upon cooking 
b. Symptoms occur within minutes or several hours after ingestion of poisonous shellfish 
c. It is caused by a plankton 
d. Symptoms include tingling sensation and paresthesia 
83. Nurse Bellatrix is giving health education to the family of Lord Voldemort regarding Red Tide Poisoning. Lord 
Voldemort asked the nurse, “Does cooking the shellfish make it safe to eat?” Nurse Bellatrix's best response is: 
a. “Yes, as long as you cook the shellfish properly at a boiling point.” 
b. “No, the poisons are not destroyed by cooking.” 
c. “Yes, the toxins are easily destroyed by heat.”
d. “No, the toxins are destroyed by freezing.” 
84. The mother of Lord Voldemort asked Nurse Bellatrix, “Which sea foods can transmit red tide poisoning to 
human?” The nurse response included the following sea foods, except: 
a. crab meat c. clams and mussels 
b. oysters and scallops d. sea cucumber 
85. Management of Red Tide Poisoning includes: 
a. Do not induce vomiting 
b. Giving of medications 
c. Give coconut milk and sodium bicarbonate 
d. The toxin is totally destroyed upon cooking 
Situation 18. Severus was admitted with manifestation of jaundice accompanied by pruritus and urticaria. The patient 
was diagnosed with Hepatitis A. 
86. Hepatitis A is also known by the following names, except: 
a. Infectious hepatitis c. Epidemic hepatitis 
b. Catarrhal jaundice d. Inoculation hepatitis 
87. Severus asked the nurse, “Why don't you give me some medication to help me get rid of this problem?” Nurse 
Neville's best response would be: 
a. “Sedatives can be given to help you relax.” 
b. “We can give you immune serum globulin.” 
c. “There are no specific drugs used to treat hepatitis.” 
d. “Vitamin supplements are frequently helpful and hasten recovery.” 
88. Nurse Neville has instructed Severus about the type of diet should be eaten. The lunch selection that would 
indicate the client's understanding and compliance with the dietary principles taught is: 
a. Turkey salad, French fries and sherbet 
b. Salad, sliced chicken sandwich and gelatin dessert 
c. Cheeseburger, taco chips and chocolate pudding 
d. Cottage cheese, peanut butter sandwich and milk shake 
89. The cooked food most likely to remain contaminated by the virus that causes Hepatitis A is which of the 
following? 
a. Canned tuna c. Fried tuna belly 
b. Broiled shrimp d. Steamed lobster 
90. Which of the following signs and symptoms occur in a patient with Hepatitis A? 
1. Loss of appetite 
2. Enlargement of the lymph nodes 
3. Tachycardia 
4. Jaundice 
5. Bradycardia 
6. Urticaria 
7. Clay-colored stools 
a. 1, 2, 4, 6 and 7 c. All except 5 
b. 2, 3, 4, and 6 d. All of the above 
Situation 19. During the rainy season, several areas in Davao City becomes flooded with water. Leptospirosis is one 
infection that is common during this time of year. 
91. Leptospirosis is also known with the following names, except: 
a. Mud fever and Flood fever 
b. Weil's disease and Trench fever 
c. Catarrhal jaundice and Ragpicker disease 
d. Spiroketal jaundice and Japanese Seven Days Fever 
92. Dobby, who is suspected of having Leptospirosis, should have his urine be tested for the presence of the 
microorganism: 
a. During the febrile period 
b. During the first week of the illness
c. During the first six months after contracting the infection 
d. After the 10th day of illness 
93. Leptospirosis can be transmitted through: 
a. Open skin contact c. Droplets 
b. Airborne d. Sexually transmitted 
94. Albus asked if it is possible to be infected while swimming in public pools or rivers. Nurse Hermione replied that: 
a. “This is not possible as long as you have no open wounds while swimming in public pools.” 
b. “Infection with leptospirosis is possible with recreational swimming in water contaminated with urine of human 
and animals having the infection.” 
c. “You can protect yourself by applying 70% alcohol before swimming 
d. “There is no danger during day time because leptospires die when exposed to sunlight. 
95. Management and treatment of Leptospirosis is symptomatic but a drug medication is prescribed at 200 mg orally 
once a week for prevention. What is the drug of choice? 
a. Chloramphenicol c. Zidovudine 
b. Doxycycline d. Streptomycin 
Situation 20. Rabies remains a public health problem in the Philippines. Approximately 300 to 600 Filipinos die of 
rabies every year. The Philippines has one of the highest prevalence rates of rabies in the whole world. 
96. Dolores Umbridge, while on her way home from work, was bitten by a stray dog. Nurse Sirius asked Dolores what 
she did immediately after the incident. Dolores's response indicates correct understanding of first aid measure when 
she says that: 
a. I went to the hospital and reported the incident 
b. I ran after the dog and had bitten it too 
c. I washed my wound immediately with soap and water 
d. I went to the drug store and bought 500 mg antibiotics 
97. Nurse Sirius interviewed Dolores and her mother at the Emergency Room to take the history before seeing a 
doctor. The nurse told Dolores's mother not to kill the dog because the dog will be: 
a. Injected with a vaccine 
b. Confined at a veterinary clinic 
c. Observed for 14 days 
d. Treated with antibiotics against rabies 
98. It is the objective of the Department of Health (DOH) to reduce the incidence of Human Rabies from 7 per million 
to 1 per million population by: 
a. 201 b. 2015 c. 2020 d. 2025 
99. Considering that rabies is one of the most acutely fatal infections which causes the death of approximately 300 to 
600 Filipinos annually, voluntary pre-exposure prophylaxis among people who are at risk is highly recommended. 
These people include: 
a. Barangay tanod officials patrolling the streets at night, side walk vendors, veterinarians and animal handlers 
b. All people living within an area where there are dogs that roam their neighborhood 
c. Pet owners, animal handlers, health personnel working in anti-rabies units and children below 15 years old 
d. Pregnant women, side walk vendors, pet owners, veterinarians and infants 
100. What is the most common caused of death in patients with Rabies? 
a. Seizures c. Hemorrhage 
b. Respiratory Paralysis d. Hydrophobia 
answers---------------------------------------------------------------- 
61. Answer: C 
Rationale: The following are late signs and symptoms of Leprosy: (3) Inability to close eyelids (lagophthalmos), 
clawaing of the fingers and toes, contractures, (6) loss of eyebrow (madarosis), enlargement of the breast in males 
(gynecomastia), chronic ulcers and (8) sinking of the nose bridge.
62. Answer: A Rationale: The mode of transmission of leprosy are the following: Airborne (inhalation of droplet/spray 
from coughing and sneezing of untreated leprosy patient) and Prolonged skin-to-skin contact. 
63. Answer: D Rationale: Multidrug Therapy is the method of treatment for leprosy. The daily treatment prescribed 
for a Multibacillary (lepromatous and borderline) type in 28 days is Clofazimine 50 mg and Dapsone 100 mg. 
64. Answer: B Rationale: Slit Skin Smear (SSS) examination is an optional procedure done only when a clinical 
diagnosis is doubtful. This prevent misclassification and wrong treatment. Options A and D is used for diagnosing 
Tuberculosis. Option C is used in Malaria. 
65. Answer: A Rationale: Bacille Calmette-Guérin (BCG) vaccine is used as preventive measure for Tuberculosis. It can 
also prevent the occurrence of Leprosy since the two disease both came from the Mycobacterium family. 
66. Answer: D Rationale: Rubeola or Measles is caused by a filterable virus (1). It is an acute highly communicable 
infection (4) characterized by fever, rashes and symptoms referable to upper respiratory tract; the eruption is 
preceded by about 2 days of coryza, during which stage grayish pecks (Koplik spots) may be found on the inner 
surface of the cheeks. A morbilliform rash appears on the 3rd or 4th day (8) affecting the face - usually beginning 
behind the ears and neck (5), body and extremities ending in branny desquamation. Death is due to complication (6) 
such as secondary pneumonia, usually in children under 2 years old 
67. Answer: A Rationale: The incubation period of Rubeola is 10 days from exposure to appearance of fever and about 
14 days until rash appears. Option B is for Chicken pox. Option C is for Mumps. Option D is for Tetanus. 
68. Answer: C Rationale: Koplik spots are tiny grayish to whitish spots found in the buccal mucosa of a client with 
Rubeola. Option A is the pathognomonic sign of Diphtheria. Option B is found in clients with Typhoid fever. Option D 
is found in clients with Rubella 
69. Answer: B Rationale: The characteristic signs of measles are which of the following: Rashes which spread from the 
face to the trunk and limbs ending in branny desquamation, conjunctivitis (Stimson's sign), high fever and tiny white 
spots in the mucosa inside the cheek 
70. Answer: B Rationale: Clients with Rubeola has conjunctivitis (Stimson's sign), as a result an important nursing 
intervention is to protect the eyes of the client from glare of strong light as they are apt to be inflamed. Place the 
client in a dim-lighted room, not well lighted. 
71. Answer: D Rationale: Typhoid fever is characterized by continued fever, anorexia (loss of appetite), slow pulse, 
involvement of lymphoid tissues, especially ulceration of Peyer's patches, enlargement of spleen, rose spots on trunk 
and abdomen and diarrhea. 
72. Answer: B Rationale: Typhoid fever is caused by Salmonella typhosa or typhoid bacillus, which is a bacteria. 
73. Answer: C Rationale: Typhoid fever can be transmitted through the following mode of transmission: direct or 
indirect contact with patient or carrier; contaminated food and water with flies as vectors; and improper food 
handling. It is not transmitted via airborne. 
74. Answer: A Rationale: Typhoid fever is characterized by continued fever, anorexia (loss of appetite), slow pulse, 
involvement of lymphoid tissues, especially ulceration of Peyer's patches, enlargement of spleen, rose spots on trunk 
and abdomen and diarrhea. (7) Rice watery stools are seen in patients with Cholera. 
75. Answer: D 
Rationale: Preventive control measures include sanitary disposal of human feces and maintenance of fly proof latrine 
and proper food handling and preparation. 
76. Answer: A Rationale: Antihelminthic drugs used in treating parasitic infections include: Albendazole and 
Mebendazole (Vermox) that inhibits glucose and other nutrient uptake of helminth; Pyrantel embonate (Antiminth) 
that paralyzes intestinal tract of worm; and Thiabendazole (Mintezol) that interferes with parasitic metabolism. 
Metronidazole (Flagyl) is an anti-protozoan drug (options B and C). Cotrimoxazole (Bactrim) is an antibiotics (option 
C). 
77. Answer: D Rationale: Commonly patients with Paragonimiasis are misdiagnosed to have Pulmonary Tuberculosis 
(PTB) and are treated as Tuberculosis patients. In fact, a study by Belizario et.al revealed that 56% of his subjects were 
non-responsive to multi-drug therapy for PTB but were positive for Paragonimiasis.
78. Answer: C Rationale: The treatment of choice for Paragonimiasis is Praziquantel (Biltrizide) 25 mg/kg body weight 
three times daily for three days. It is suitable for treatment of adults and children over four years of age, higher dose 
is needed for ectopic paragonimiasis. Bithionol (BITIN) is only an alternative drug for paragonimiasis (option A). 
Ethambutol (option B) is one of the multi-drug therapy for PTB. Metronidazole (option D) is an anti-protozoan drug. 
79. Answer: B Rationale: The three major causes of intestinal parasitic infections in the Philippines are the following: 
(A) Trichuriasis or Whipworm (Trichuris trichiura); (C) Ancylostomiasis or Hookworm (Ancylostoma duodenale and 
Necator americanus); and (D) Ascariasis (Ascaris lumbricoides). 
80. Answer: D Rationale: Pinworms (enterobiasis), which is caused by the parasite Enterobius vermicularis, is the most 
common helminthic infestation among children. It enters the intestinal mucosa and could be spread through 
autoinfection (when the child places fingers, used in scratching the anal part and containing the pinworm eggs, into 
the mouth). It is noted that pinworms caused severe itching as they lay their eggs. 
81. Answer: c Rationale: The signs and symptoms of Red Tide Poisoning includes: numbness of the face especially 
around the mouth; vomiting; dizziness; headache; tingling sensation, paresthesia and eventual paralysis of hands and 
feet; Floating sensation and weakness; rapid pulse (tachycardia); difficulty of speech (ataxia); difficulty of swallowing 
(dysphagia); and total muscle paralysis with respiratory arrest and death occur in severe cases. 
82. Answer: A Rationale: Paralytic Shellfish Poisoning (PCP) or Red Tide Poisoning is a syndrome of characteristic 
symptoms predominantly neurologic which occur within minutes or several hours after ingestion of poisonous 
shellfish (option B). It is caused by a single celled organism called dinoflagellates, which is commonly referred to as 
plankton (option C). One of its symptoms include tingling sensation or paresthesia (option D). Toxins in red tide is not 
totally destroyed upon cooking. 
83. Answer: B Rationale: Toxins in red tide is not totally destroyed upon cooking hence consumers must be educated 
to avoid bi-valve mollusks like tahong, talaba, halaan, kabiya, abaniko (sun and moon shell or Asian scallop) when the 
red tide warning has been issued by the proper authorities. 
84. Answer: D Rationale: Bi-valve mollusks such as oysters, clams, mussels, sun and moon shell or Asian scallop as well 
as crabs are avoided when the red tide warning has been issued by the proper authorities. There's no reported cases 
of red tide poisoning when eating sea cucumber. 
85. Answer: C Rationale: Management of Red Tide Poisoning includes the following: induce vomiting; no definite 
medication indicated; drinking pure coconut milk, which weakens the toxic effect of red tide, and sodium bicarbonate 
solution (25 grams in ½ glass of water) may be taken. Drinking coconut milk and sodium bicarbonate solution is 
advised during the early stages of poisoning only. If given during the late stag, they may make the condition of the 
patient worse; Shellfish affected by red tide must not be cooked with vinegar as the toxin of Pyromidium increases 
(15 time greater) when mixed with acid; and do not eat even cooked shellfish because toxins of red tide are not 
totally destroyed upon cooking. 
86. Answer: D 
Rationale: The other names of Hepatitis A are the following: Infectious hepatitis; Catarrhal jaundice; and Epidemic 
hepatitis. In option D, Inoculation hepatitis is the other name for Hepatitis B. 
87. Answer: C Rationale: There are no specific drugs for the treatment of Hepatitis A. There is, however, a 
prophylaxis, which is an IM injection of gamma globulin. 
88. Answer: B Rationale: The dietary requirement for a client with Hepatitis A is a low fat diet but high in sugar. 
Options A, C and D are incorrect because all or part of its choices are high in fats. Low fat diet include only lean meats, 
fish and poultry. The allowed vegetable oils may be used in preparing meats, fish and poultry; used in salad dressings; 
or in baked products. 
89. Answer: D Rationale: Hepatitis A virus can be killed by cooking the food to boiling point, steaming the food will 
not kill the virus. 
90. Answer: A 
Rationale: Hepatitis A has the following signs and symptoms: influenza-like such as headache; malaise and easy 
fatigability; loss of appetite or anorexia (1); abdominal discomfort/pain; nausea and vomiting; fever; enlargement of 
the lymph node or lymphadenopathy (2); jaundice (4) accompanied by pruritus and urticaria (6); and bilirubinemia 
with clay-colored stools (7). 
91. Answer: C
Rationale: Leptospirosis is also known with the following names: Mud fever and Flood fever (option A); Weil's disease 
and Trench fever (option B); and Spiroketal jaundice and Japanese Seven Days Fever (option D). In option C, Catarrhal 
jaundice is the other name for Hepatitis A while Ragpicker disease is the other name for Anthrax. 
92. Answer: D 
Rationale: Leptospirosis can be diagnosed by its clinical manifestations, culture of the organism, examination of 
blood and CSF during the first week of illness and urine after the 10th day. 
93. Answer: A 
Rationale: Leptospirosis can be transmitted through contact of the skin, especially open wounds with water, moist 
soil or vegetation contaminated with urine of infected host. 
94. Answer: B 
Rationale: Infection with leptospirosis is possible with recreational swimming in water contaminated with urine of 
human and animals having the infection. 
95. Answer: B 
Rationale: Doxycycline is the drug of choice for Leptospirosis. It is prescribed at 200 mg orally once a week for 
prevention 
96. Answer: C 
Rationale: The wound must be immediately and thoroughly washed with soap and water. Antiseptics such as 
povidone iodine or alcohol may be applied. 
97. Answer: C 
Rationale: Patients should consult a veterinarian or trained personnel to observe the pet for 14 days for signs of 
rabies. 
98. Answer: A 
Rationale: The general objective of the Department of Health (DOH) is to reduce the incidence of Human Rabies from 
7 per million to 1 per million population by 2010 and eliminate human rabies by 2015. 
99. Answer: C 
Rationale: People who are at high risk of acquiring rabies includes the following: Pet owners; animal handlers; health 
personnel working in anti-rabies units; and children below 15 years old. 
100. Answer: B 
Rationale: Without medical intervention, the rabies victim would usually last only for 2 to 6 days. Death is often due 
to respiratory paralysis. 
Question------------------------------------- 
Situation 7: Ralph, a student nurse began to feel joint pains and eye pains associated with high fever. He noticed 
some pinpoint rashes on his legs. He is diagnosed with Dengue fever. 
31. In the prevention of dengue fever, which of the following measures is included? 
a. Mass vaccination during rainy season 
b. Prophylactic drug treatment with antivirals 
c. Removal of small collections of water such as flower vases 
d. Fogging or daily spraying of insecticides 
32. The most effective control measure for dengue hemorrhagic fever is: 
a. Isolation of patient 
b. Mosquito control 
c. Health education 
d. Concurrent disinfection 
33. The treatment and control of dengue hemorrhagic fever include all of the following EXCEPT: 
a. Treat shock with IVF 
b. Encourage vaccination 
c. Eradication of vector 
d. Community participation 
34. The best nursing intervention in the management of dengue fever among patients admitted in the hospital would 
be:
a. Observation 
b. Contact Isolation 
c. Administration of antivirals 
d. Platelet administration 
35. Which is an independent nursing intervention for a patient with Dengue Hemorrhagic Fever? 
a. Give aspirin for fever and muscle pains 
b. Put ice bag over forehead if patient has nose bleed 
c. Place patient in prone position to relieve abdominal pain 
d. Give high fiber diet 
Situation 8: The BHW of Barangay Jacinto reported series of adults having cases of spasmodic coughing and fever for 
the past two weeks. The CHN in the area alarmed by the reports decided to investigate the community. 
36. Which among the following diagnostic tests does NOT indicate active tuberculosis? 
a. Sputum smear positive times three 
b. Chest x-ray as diagnosed by the TBDC 
c. An induration of 10 post tuberculin test 
d. Chest x-ray after having three negative sputum smears 
37. Mang Jose, 42 years old, went to the Chest Center to avail of the free Direct Sputum Smear Microscopy. Which 
among the following is a contraindication for sputum collection? 
a. Pleural effusion c. Hemoptysis 
b. TB meningitis d. Miliary TB 
38. Mang Jose began his treatment regimen for tuberculosis. He complains of frequent tingling sensation of his feet. 
Which among the following diets shout be encouraged? 
a. Fish c. Milk 
b. Kangkong d. Beef 
39. As a nurse working in the Barangay Health Center, it is essential to counsel the client on the side effects of the 
treatment regimen. Which among the following side effects should be reported immediately to the physician? 
a. Hemoptysis c. Flu-like symptoms 
b. Red-colored urine d. Yellowish palms 
40. Which among the following is considered as the most important health teaching given to clients with 
Tuberculosis? 
a. Strict compliance to treatment regimen 
b. Frequent handwashing 
c. Proper disposal of soiled tissues 
d. Covering of mouth while coughing or sneezing 
Situation 9: The nurse is aware of the misuse of antibiotics , which resulted to the emergence of new strains of 
microorganisms that proves to effect a catastophic effect on human health. 
41. The period between the first exposure and multiplication of the SRAS causative agent would be: 
a. 2-10 days c. 1-20 days 
b. 5-11 days d. 1-5 days 
42. SARS remains to be an enigma among health practitioners and had proven its virulence. The nurse knows that the 
virus could be categorized as: 
a. Rhabdovirus c. Coronavirus 
b. Togavirus d. Paramyxovirus 
43. Which of the following best defines standards precaution? 
a. These are guidelines created to prevent transmission of microorganisms in hospitals 
b. It is a strategy of assuming that all patients are infectious, and using barrier protections during nurse-client 
interactions 
c. These are precautions based on contagious or epidemiologically significant organisms are recognized. 
d. It pertains to the use of handwashing. 
44. In managing individuals with SARS, the nurse should be aware of the different signs and symptoms of the disease. 
Which among the following characterize the prodromal phase:
a. The patient is usually not infectious at this time. 
b. Dry hacking cough is present 
c. Fever is present, but does not rise beyond 38oC 
d. The patient requires mechanical ventilation at this stage 
45. Handwashing deters the spread of microorganisms. A nurse involved in the care of a patient diagnosed with SARS 
should: 
a. Perform handwashing before client contact 
b. Perform handwashing after eating 
c. Perform handwashing after food preparation 
d. Perform handwashing before using the toilet 
Situation 10: Filariasis is endemic in some parts of the Philippines. The disease often progresses to become chronic, 
debilitating and often unfamiliar to health workers. 
46. Effective methods that the government would likely to pursue to eliminate filariasis in the country are all of the 
following EXCEPT: 
a. Pursue annual mass drug administration using two drugs in all endemic areas for at least five consecutive years 
b. Vaccination of all susceptible in high risk areas and high risk populations 
c. Intensify health information and advocacy campaigns in its prevention, control and elimination 
d. Halt progression of disease through disability prevention 
47. The vector for Filariasis is 
a. Wuchereria bancrofti 
b. Aedes poecillus 
c. Anopheles 
d. Aedes egypti 
48. A long incubation period characterizes Filariasis that typically ranges from: 
a. 2-4 weeks c. 2-3 years 
b. 4-6 weeks d. 8-16 months 
49. A 36-year-old man is brought by his wife to a doctor's clinic to be tested for filariasis. The most likely diagnostic 
test that he will undergo is: 
a. Immunochromatographic test (ICT) 
b. Nocturnal Blood Examination 
c. Stool examination 
d. Urinalysis 
50. A client in the acute stage of the disease will include which of the following clinical findings? 
a. Lymphangitis, lymphadenitis, epidydimitis 
b. Hydrocele, lymphedema, elephantiasis 
c. Orchitis, hydrocele, elephantiasis 
d. Lymphadenitis, lympedema and orchitis 
Situation 11: Specific defenses of the body involve the immune system. Nurses should be knowledgeable on the 
importance of immunity in the prevention of communicable diseases. 
51. Ayisha is given the hepatitis B immune globulin serum, which will provide her with passive immunity. One 
advantage of passive immunity is that it: 
a. Has effects that last a long time 
b. Is highly effective in the treatment of disease 
c. Offers immediate protection 
d. Encourages the body to produce antibodies 
52. Nurse Keith explains to a mother whose child just received a tetanus toxoid injection that the toxoid confers 
which of the following immunity? 
a. Lifelong passive immunity 
b. Long-lasting active immunity 
c. Lifelong active natural immunity 
d. Lifelong active artificial immunity 
53. Simon, who is suspected of having tetanus, asks Nurse Benny about immunizations against tetanus. Nurse Benny 
explains that the major benefit in using tetanus antitoxin is that it:
a. Stimulates plasma cells directly 
b. Provides a high titer of antibodies 
c. Provides immediate active immunity 
d. Stimulates long-lasting passive immunity 
54. Justin, who was exposed to hepatitis A is given gamma globulin to provide passive immunity, which: 
a. Increases production of short-lived antibodies 
b. Provides antibodies that neutralize the antigen 
c. Accelerates antigen-antibody union at the hepatic sites 
d. Stimulates the lymphatic system to produce large numbers of antibodies 
55. A mother asked Nurse Edna on the duration of the effectiveness of a natural passive immunity. Nurse Edna is 
correct when she tell the mother that the effect lasts for: 
a. 2 to 3 weeks c. 6 months to 1 year 
b. Permanent d. 2 to 5 years 
Situation 12. Whenever possible, the nurse implements strategies to prevent infection. If infection cannot be 
prevented, the nurse's goal is to prevent the spread of the infection within and between persons, and to treat the 
existing infection. 
56. Gino is a chronic carrier of infection. To prevent the spread of the infection to other clients or health care 
providers, the nurse emphasizes interventions that do which of the following? 
a. Eliminate the reservoir 
b. Block the portal of exit from the reservoir 
c. Block the portal of entry into the host 
d. Decrease the susceptibility of the host 
57. The most effective nursing action for controlling the spread of infection includes which of the following? 
a. Thorough hand cleansing 
b. Wearing gloves and masks when providing direct client care 
c. Implementing appropriate isolation precautions 
d. Administering broad-spectrum prophylactic antibiotics 
58. In caring for Mark who is on contact precautions for a draining infected foot ulcer, Nurse Bong should perform 
which of the following? 
a. Wear a mask during dressing changes 
b. Provide disposable meal trays and silverware 
c. Follow standard precautions in all interactions with the client 
d. Use surgical aseptic technique for all direct contact with the client 
59. When caring for a single client during one shift, it is appropriate for the nurse to reuse which of the following 
personal protective equipment? 
a. Goggles c. Surgical mask 
b. Gown d. Clean gloves 
60. After teaching a client and family strategies to prevent infection, which statement by the client would indicate 
effective learning has occurred? 
a. “We will use antimicrobial soap and hot water to wash our hands at least three times per day.” 
b. “We must wash or peel all raw fruits and vegetables before eating.” 
c. “A wound or sore is not infected unless we see it draining pus.” 
d. “We should not share toothbrushes but it is OK to share towels and washcloths. 
Answer----------------------------------- 
31. C. 
A and B. There is no known vaccination and prophylactic antivirals for Dengue fever. 
D. Eliminating the breeding sites of mosquitoes is better than killing the mosquitoes. 
32. B 
33. B. There is no known vaccination for Dengue fever. 
34. A. Patient should be observed closely for signs of deterioration (shock) such as hypotension, tachycardia, 
tachypnea, cold clammy perspiration and prostration. 
B. This is not necessary. 
C. There is no known antivirals for Dengue fever.
D. Platelet administration is done to prevent bleeding, but this is a dependent function. 
35. B. This promotes vasoconstriction. 
A. Aspirin is contraindicated. It may worsen the bleeding. 
C. Patient should be placed in dorsal recumbent to facilitate blood circulation. 
D. Diet should be low fiber, low fat, non-irritating, and non-carbonated. 
36. C. Tuberculin/Mantoux test only indicate exposure to the mycobacterium, not active infection. 
A and D. After obtaining three sputum negative results, chest x-ray can be done to assess parenchymal involvement. 
Diagnosis of TB through x-ray can be made by the TB 
Diagnostic Center (TBDC). 
B. Direct Sputum Smear Microscopy (DSSM) is the primary diagnostic tool for TB. 
37. C. Hemoptysis is the only contraindication for sputum collection. 
A, B and D are indications for hospitalization of clients with TB. 
38. B. The Isoniazid in the treatment regimen causes peripheral neuropathy. Client should be advised to increase 
intake of Vitamin B6, which is commonly found in green leafy vegetables, such as kangkong. 
39. D. Yellowish palms may indicate liver problem. Anti-tuberculous drugs, particularly Pyrazinamide, are hepatotoxic 
drugs. 
A. Hemoptysis is not a side effect, but must be reported immediately to the physician. Massive hemoptysis requires 
hospitalization. 
B and C are side effects of Isoniazid. Clients should be advised to increase oral fluid intake. Paracetamol is given to 
manage the fever. 
40. A. Strict compliance to the treatment regimen is a must while undergoing treatment for TB, to avoid relapses and 
prevent the development of multi-drug resistant strain of TB. 
B, C and D are part of the health teaching to clients with tuberculosis, but strict compliance to treatment regimen 
should be given utmost emphasis. 
41. A. The incubation period for SARS is 2-10 days but may be long as 13 days. 
42. C. 
A. Rhabdovirus causes rabies infection. 
B. Togavirus causes German measles. 
D. The family of paramyxovirus is responsible for measles and mumps. 
43. B. 
A. This pertain to isolation precaution. 
C. This pertain to transmission-based precaution. 
D. Handwashing is a universal precautionary measure. 
44. A. During the prodromal phase, infectivity is none to low. Infectivity is highest during the respiratory phase (within 
2-7 days). 
B and D are seen during the respiratory stage. 
C. Fever during the prodromal phase of SRAS is above 380C. 
45. A. Handwashing is performed before and after client contact 
B. Handwashing is performed before eating. 
C. Handwashing is performed before food preparation. 
D. Handwashing is performed after using the toilet. 
46. B. There is no known vaccination for Filariasis. Diethycarbamezine Citrate (DEC) is given to patients with clinical 
manifestations and/or microfilariae. 
47. B. A. This is the causative agent for filariasis. Other causative agents include Brugia malayi and Brugia timori. 
C. This is the vector for Malaria. 
D. This is the vector for Dengue. 
48. D. Incubation period ranges from 8-16 months.
49. A. The clinic is only open from 8am to 5pm. The ICT is an antigen test that can be done in daytime. Nocturnal 
blood smear is also a diagnostic test for Filariasis, but the patient's blood is taken at the patient's residence or in the 
hospital after 8pm. 
50. A. The clinic is only open from 8am to 5pm. The ICT is an antigen test that can be done in daytime. Nocturnal 
blood smear is also a diagnostic test for Filariasis, but the patient's blood is taken at the patient's residence or in the 
hospital after 8pm. 
51. A. Lymphangitis, lymphadenitis, epidydimitis, funiculitis and orchotis are acute clinical manifestations of Filariasis. 
Hydrocele, lymphedema, and elephantiasis are clinical manifestations of Chronic Filariasis. 
52. Answer: C Rationale: Passive immunity provides immediate protection, it is short-lived, is limited in effectiveness, 
and does not stimulate the body to produce antibodies. (NSNA NCLEX-RN Review, 4th Edition) 
53. Answer: B 
Rationale: Tetanus antitoxin provides antibodies, which confer immediate passive artificial immunity, the same with 
antiserums and immune globulins. Option A is incorrect because antitoxins doesn't stimulate production of 
antibodies. Option C is incorrect because it provides passive, not active, immunity. Option D is incorrect because 
passive immunity, by definition, is not long-lasting. (Mosby's, 18th Edition) 
54. Answer: B 
Rationale: Gamma globulin, an immune globulin, contains most of the antibodies circulating in the blood. When 
injected into an individual, it prevents a specific antigen from entering a host cell. Options A and D are incorrect 
because this does not stimulate antibody production. Option C is incorrect because this does not affect antigen-antibody 
function. (Mosby's, 18th Edition) 
55. Answer: C 
Rationale: The effectiveness of a natural passive immunity lasts for 6 months to 1 year. Option B is for natural active 
immunity. Option A is for artificial passive immunity. Option C is for artificial active immunity. 
56. Answer: B 
Rationale: Blocking the movement of the organism from the reservoir will succeed in preventing the infection of any 
other persons. In option A, since the carrier person is the reservoir and the condition is chronic, it is not possible to 
eliminate the reservoir. Option C and D are incorrect because blocking the entry into a host or decreasing the 
susceptibility of the host will be effective for only that one single individual and, thus, is not effective as blocking exit 
from the reservoir. 
57. Answer: A Rationale: Since the hands are frequently in contact with clients and equipment, they are the most 
obvious source of transmission. Regular and routine hand cleansing is the most effective way to prevent movement of 
potentially infective materials. Option B is incorrect because personal protective equipment (PPE) such as gloves and 
masks is indicated for situations requiring Standard Precautions. Option C is incorrect because Isolation precautions 
are used for clients with known communicable diseases. Option D is incorrect because use of antibiotics is not 
effective and can be harmful due to the incidence of superinfection and development of resistant organisms. 
58. Answer: C Rationale: Standard Precautions include all aspects of contact precautions with the exception of placing 
the client in a private room. In option A, a mask is indicated when working over a sterile wound rather than an 
infected one. Option B is incorrect because disposable food trays are not necessary for clients with infected wounds 
unlikely to contaminate the client's handstion D is incorrect because sterile technique (surgical asepsis) is not 
indicated for all contact with the client. The nurse would utilize clean technique when dressing the wound to prevent 
introduction of additional microbes. 
59. Answer: A Rationale: Unless overly contaminated by material that has splashed in the nurse's face and cannot be 
effectively rinsed off, goggles may be worn repeatedly. In option B, since gowns are at high risk for contamination, 
they should be used only once and hen discarded or washed. Option C and D are incorrect because surgical mask and 
clean gloves are never washed or reused. 
60.Answer: B Rationale: Raw foods touched by human hands can carry significant infectious organisms and must be 
washed or peeled. Option A is incorrect because antimicrobial soap is not indicated for regular use and may lead to 
resistant organisms. Hand cleansing should occur as needed. Hot water can dry and harm skin, increasing the risk of 
infection. In option C, clients should learn all the signs of inflammation and infection (e.g., redness, swelling, pain and 
heat) and not rely on the presence of pus to indicate this. Option D is incorrect because persons should not share 
washcloths or towels.

More Related Content

What's hot

emergency-nursing
 emergency-nursing emergency-nursing
emergency-nursingAnjo Ortiz
 
MCQ. Medical-surgical Nursing
MCQ. Medical-surgical NursingMCQ. Medical-surgical Nursing
MCQ. Medical-surgical NursingNursing Path
 
50 mcqs on community health nursing part 1
50 mcqs on community health nursing part 150 mcqs on community health nursing part 1
50 mcqs on community health nursing part 1ManimegalaiSiva
 
Answered national council exam mixed 1-2.docx
Answered national council exam mixed 1-2.docxAnswered national council exam mixed 1-2.docx
Answered national council exam mixed 1-2.docxRBGroup
 
Maternal and child health nursing
Maternal and child health nursingMaternal and child health nursing
Maternal and child health nursingRuby Shelah Dunque
 
Physical assessment quiz dr. khaled khader
Physical assessment quiz   dr. khaled khaderPhysical assessment quiz   dr. khaled khader
Physical assessment quiz dr. khaled khaderkhaledkhader
 
Focus Charting (FDAR)
Focus Charting (FDAR)Focus Charting (FDAR)
Focus Charting (FDAR)Jack Frost
 
8. cesarean section
8. cesarean section8. cesarean section
8. cesarean sectionHishgeeubuns
 
Impaired Gas Exchange Nursing Care Plan
Impaired Gas Exchange  Nursing Care PlanImpaired Gas Exchange  Nursing Care Plan
Impaired Gas Exchange Nursing Care PlanNursing for Life
 
PNLE Medical Surgical Nursing Exam 1.docx
PNLE Medical Surgical Nursing Exam 1.docxPNLE Medical Surgical Nursing Exam 1.docx
PNLE Medical Surgical Nursing Exam 1.docxAbdhaniePanontongan
 
2012 NATIONAL NURSING CORE COMPETENCY STANDARDS
2012 NATIONAL NURSING CORE COMPETENCY STANDARDS2012 NATIONAL NURSING CORE COMPETENCY STANDARDS
2012 NATIONAL NURSING CORE COMPETENCY STANDARDSHanna Olvido
 
Fundamental of nursing practice exam 4
Fundamental of nursing practice exam 4Fundamental of nursing practice exam 4
Fundamental of nursing practice exam 4Nursing Path
 
Nursing management quiz
Nursing management quizNursing management quiz
Nursing management quizNursing Path
 

What's hot (20)

Nepal nursing council licencing exam mcqs sample
Nepal nursing council licencing exam mcqs sampleNepal nursing council licencing exam mcqs sample
Nepal nursing council licencing exam mcqs sample
 
CHN EXAM.pdf
CHN EXAM.pdfCHN EXAM.pdf
CHN EXAM.pdf
 
MCQs for Entrance Test for BN, MN, MSN Nursing by RS MEHTA
MCQs for Entrance Test for BN, MN, MSN Nursing by RS MEHTAMCQs for Entrance Test for BN, MN, MSN Nursing by RS MEHTA
MCQs for Entrance Test for BN, MN, MSN Nursing by RS MEHTA
 
emergency-nursing
 emergency-nursing emergency-nursing
emergency-nursing
 
MCQ. Medical-surgical Nursing
MCQ. Medical-surgical NursingMCQ. Medical-surgical Nursing
MCQ. Medical-surgical Nursing
 
50 mcqs on community health nursing part 1
50 mcqs on community health nursing part 150 mcqs on community health nursing part 1
50 mcqs on community health nursing part 1
 
Answered national council exam mixed 1-2.docx
Answered national council exam mixed 1-2.docxAnswered national council exam mixed 1-2.docx
Answered national council exam mixed 1-2.docx
 
Maternal and child health nursing
Maternal and child health nursingMaternal and child health nursing
Maternal and child health nursing
 
Physical assessment quiz dr. khaled khader
Physical assessment quiz   dr. khaled khaderPhysical assessment quiz   dr. khaled khader
Physical assessment quiz dr. khaled khader
 
Focus Charting (FDAR)
Focus Charting (FDAR)Focus Charting (FDAR)
Focus Charting (FDAR)
 
8. cesarean section
8. cesarean section8. cesarean section
8. cesarean section
 
Impaired Gas Exchange Nursing Care Plan
Impaired Gas Exchange  Nursing Care PlanImpaired Gas Exchange  Nursing Care Plan
Impaired Gas Exchange Nursing Care Plan
 
PNLE Medical Surgical Nursing Exam 1.docx
PNLE Medical Surgical Nursing Exam 1.docxPNLE Medical Surgical Nursing Exam 1.docx
PNLE Medical Surgical Nursing Exam 1.docx
 
Ncp of copd
Ncp of copdNcp of copd
Ncp of copd
 
2012 NATIONAL NURSING CORE COMPETENCY STANDARDS
2012 NATIONAL NURSING CORE COMPETENCY STANDARDS2012 NATIONAL NURSING CORE COMPETENCY STANDARDS
2012 NATIONAL NURSING CORE COMPETENCY STANDARDS
 
Fundamental of nursing practice exam 4
Fundamental of nursing practice exam 4Fundamental of nursing practice exam 4
Fundamental of nursing practice exam 4
 
Nursing management quiz
Nursing management quizNursing management quiz
Nursing management quiz
 
Care of the Newborn Handouts
Care of the Newborn  HandoutsCare of the Newborn  Handouts
Care of the Newborn Handouts
 
Medical Surgical Nursing
Medical Surgical NursingMedical Surgical Nursing
Medical Surgical Nursing
 
Asthma
AsthmaAsthma
Asthma
 

Similar to Communicable disease q&a

Communicable disease ans. key
Communicable disease ans. keyCommunicable disease ans. key
Communicable disease ans. keyFern Mosquera
 
Krok 1 - 2015 (Microbiology)
Krok 1 - 2015 (Microbiology)Krok 1 - 2015 (Microbiology)
Krok 1 - 2015 (Microbiology)Eneutron
 
COC based excercises for health professionals
COC based excercises for health professionalsCOC based excercises for health professionals
COC based excercises for health professionalsSamuel Dessu
 
Coc based excercises
Coc based excercisesCoc based excercises
Coc based excercisesSamuel Dessu
 
Krok 1 - 2014 (Microbiology)
Krok 1 -  2014 (Microbiology)Krok 1 -  2014 (Microbiology)
Krok 1 - 2014 (Microbiology)Eneutron
 
council exams 2015- correction.docx
council exams 2015- correction.docxcouncil exams 2015- correction.docx
council exams 2015- correction.docxRBGroup
 
STEP 2 - Nums past paper by dr. shahid alam
STEP 2 - Nums past paper by dr. shahid alamSTEP 2 - Nums past paper by dr. shahid alam
STEP 2 - Nums past paper by dr. shahid alamDr. Shadab Kamal
 
Obs and gynae data bank
Obs and gynae data bankObs and gynae data bank
Obs and gynae data bankwiseman chanda
 
Fundamental of nursing quiz
Fundamental of nursing quizFundamental of nursing quiz
Fundamental of nursing quizNursing Path
 
Nclex test review
Nclex test reviewNclex test review
Nclex test reviewmerris35
 
2017留学生试卷-2021-2022-2线上试题-A卷.pdf
2017留学生试卷-2021-2022-2线上试题-A卷.pdf2017留学生试卷-2021-2022-2线上试题-A卷.pdf
2017留学生试卷-2021-2022-2线上试题-A卷.pdfSamikshyaKharel
 
Sexually Transmitted Infections
Sexually Transmitted InfectionsSexually Transmitted Infections
Sexually Transmitted InfectionsEric Meininger
 

Similar to Communicable disease q&a (20)

Communicable disease ans. key
Communicable disease ans. keyCommunicable disease ans. key
Communicable disease ans. key
 
Krok 1 - 2015 (Microbiology)
Krok 1 - 2015 (Microbiology)Krok 1 - 2015 (Microbiology)
Krok 1 - 2015 (Microbiology)
 
Chn2
Chn2Chn2
Chn2
 
COC based excercises for health professionals
COC based excercises for health professionalsCOC based excercises for health professionals
COC based excercises for health professionals
 
Coc based excercises
Coc based excercisesCoc based excercises
Coc based excercises
 
Krok 1 - 2014 (Microbiology)
Krok 1 -  2014 (Microbiology)Krok 1 -  2014 (Microbiology)
Krok 1 - 2014 (Microbiology)
 
council exams 2015- correction.docx
council exams 2015- correction.docxcouncil exams 2015- correction.docx
council exams 2015- correction.docx
 
STEP 2 - Nums past paper by dr. shahid alam
STEP 2 - Nums past paper by dr. shahid alamSTEP 2 - Nums past paper by dr. shahid alam
STEP 2 - Nums past paper by dr. shahid alam
 
Obs and gynae data bank
Obs and gynae data bankObs and gynae data bank
Obs and gynae data bank
 
Comprhensive
ComprhensiveComprhensive
Comprhensive
 
Fundamental of nursing quiz
Fundamental of nursing quizFundamental of nursing quiz
Fundamental of nursing quiz
 
Gd oor 912
Gd oor 912Gd oor 912
Gd oor 912
 
Mcqs food borne
Mcqs food borneMcqs food borne
Mcqs food borne
 
Nclex test review
Nclex test reviewNclex test review
Nclex test review
 
Drill 3
Drill 3Drill 3
Drill 3
 
Mcq ped neuro
Mcq ped neuroMcq ped neuro
Mcq ped neuro
 
2017留学生试卷-2021-2022-2线上试题-A卷.pdf
2017留学生试卷-2021-2022-2线上试题-A卷.pdf2017留学生试卷-2021-2022-2线上试题-A卷.pdf
2017留学生试卷-2021-2022-2线上试题-A卷.pdf
 
1
11
1
 
MCQ-PEDIA.docx
MCQ-PEDIA.docxMCQ-PEDIA.docx
MCQ-PEDIA.docx
 
Sexually Transmitted Infections
Sexually Transmitted InfectionsSexually Transmitted Infections
Sexually Transmitted Infections
 

More from iteach 2learn

Overview of Philippine educational system and its present status
Overview of Philippine educational system  and its present statusOverview of Philippine educational system  and its present status
Overview of Philippine educational system and its present statusiteach 2learn
 
Consent deworming- "kontra bulate"
Consent deworming- "kontra bulate"Consent deworming- "kontra bulate"
Consent deworming- "kontra bulate"iteach 2learn
 
Overview on writing research problem,introduction & background of the study
Overview on writing research problem,introduction & background of the studyOverview on writing research problem,introduction & background of the study
Overview on writing research problem,introduction & background of the studyiteach 2learn
 
Anti bullying Act 2013 & DO no. 40 s. 2012
Anti bullying Act 2013 & DO no. 40 s. 2012Anti bullying Act 2013 & DO no. 40 s. 2012
Anti bullying Act 2013 & DO no. 40 s. 2012iteach 2learn
 
Earthquake safety in schools (Philippines)
Earthquake safety in schools (Philippines)Earthquake safety in schools (Philippines)
Earthquake safety in schools (Philippines)iteach 2learn
 
Grade 8 science teacher's guide
Grade 8 science teacher's guideGrade 8 science teacher's guide
Grade 8 science teacher's guideiteach 2learn
 
Code of ethics for professional teachers Art 7
Code of ethics for professional teachers Art 7Code of ethics for professional teachers Art 7
Code of ethics for professional teachers Art 7iteach 2learn
 
Focus Charting adapted ZCMC Pedia
Focus Charting adapted ZCMC PediaFocus Charting adapted ZCMC Pedia
Focus Charting adapted ZCMC Pediaiteach 2learn
 
Health Assessment in Nursing
Health Assessment in NursingHealth Assessment in Nursing
Health Assessment in Nursingiteach 2learn
 

More from iteach 2learn (10)

Overview of Philippine educational system and its present status
Overview of Philippine educational system  and its present statusOverview of Philippine educational system  and its present status
Overview of Philippine educational system and its present status
 
Consent deworming- "kontra bulate"
Consent deworming- "kontra bulate"Consent deworming- "kontra bulate"
Consent deworming- "kontra bulate"
 
Overview on writing research problem,introduction & background of the study
Overview on writing research problem,introduction & background of the studyOverview on writing research problem,introduction & background of the study
Overview on writing research problem,introduction & background of the study
 
Anti bullying Act 2013 & DO no. 40 s. 2012
Anti bullying Act 2013 & DO no. 40 s. 2012Anti bullying Act 2013 & DO no. 40 s. 2012
Anti bullying Act 2013 & DO no. 40 s. 2012
 
Earthquake safety in schools (Philippines)
Earthquake safety in schools (Philippines)Earthquake safety in schools (Philippines)
Earthquake safety in schools (Philippines)
 
Bahagi ng pahayagan
Bahagi ng pahayaganBahagi ng pahayagan
Bahagi ng pahayagan
 
Grade 8 science teacher's guide
Grade 8 science teacher's guideGrade 8 science teacher's guide
Grade 8 science teacher's guide
 
Code of ethics for professional teachers Art 7
Code of ethics for professional teachers Art 7Code of ethics for professional teachers Art 7
Code of ethics for professional teachers Art 7
 
Focus Charting adapted ZCMC Pedia
Focus Charting adapted ZCMC PediaFocus Charting adapted ZCMC Pedia
Focus Charting adapted ZCMC Pedia
 
Health Assessment in Nursing
Health Assessment in NursingHealth Assessment in Nursing
Health Assessment in Nursing
 

Recently uploaded

Call Girl Service Bidadi - For 7001305949 Cheap & Best with original Photos
Call Girl Service Bidadi - For 7001305949 Cheap & Best with original PhotosCall Girl Service Bidadi - For 7001305949 Cheap & Best with original Photos
Call Girl Service Bidadi - For 7001305949 Cheap & Best with original Photosnarwatsonia7
 
High Profile Call Girls Jaipur Vani 8445551418 Independent Escort Service Jaipur
High Profile Call Girls Jaipur Vani 8445551418 Independent Escort Service JaipurHigh Profile Call Girls Jaipur Vani 8445551418 Independent Escort Service Jaipur
High Profile Call Girls Jaipur Vani 8445551418 Independent Escort Service Jaipurparulsinha
 
Russian Call Girl Brookfield - 7001305949 Escorts Service 50% Off with Cash O...
Russian Call Girl Brookfield - 7001305949 Escorts Service 50% Off with Cash O...Russian Call Girl Brookfield - 7001305949 Escorts Service 50% Off with Cash O...
Russian Call Girl Brookfield - 7001305949 Escorts Service 50% Off with Cash O...narwatsonia7
 
Call Girls Electronic City Just Call 7001305949 Top Class Call Girl Service A...
Call Girls Electronic City Just Call 7001305949 Top Class Call Girl Service A...Call Girls Electronic City Just Call 7001305949 Top Class Call Girl Service A...
Call Girls Electronic City Just Call 7001305949 Top Class Call Girl Service A...narwatsonia7
 
Call Girls Jayanagar Just Call 7001305949 Top Class Call Girl Service Available
Call Girls Jayanagar Just Call 7001305949 Top Class Call Girl Service AvailableCall Girls Jayanagar Just Call 7001305949 Top Class Call Girl Service Available
Call Girls Jayanagar Just Call 7001305949 Top Class Call Girl Service Availablenarwatsonia7
 
97111 47426 Call Girls In Delhi MUNIRKAA
97111 47426 Call Girls In Delhi MUNIRKAA97111 47426 Call Girls In Delhi MUNIRKAA
97111 47426 Call Girls In Delhi MUNIRKAAjennyeacort
 
call girls in Connaught Place DELHI 🔝 >༒9540349809 🔝 genuine Escort Service ...
call girls in Connaught Place  DELHI 🔝 >༒9540349809 🔝 genuine Escort Service ...call girls in Connaught Place  DELHI 🔝 >༒9540349809 🔝 genuine Escort Service ...
call girls in Connaught Place DELHI 🔝 >༒9540349809 🔝 genuine Escort Service ...saminamagar
 
Case Report Peripartum Cardiomyopathy.pptx
Case Report Peripartum Cardiomyopathy.pptxCase Report Peripartum Cardiomyopathy.pptx
Case Report Peripartum Cardiomyopathy.pptxNiranjan Chavan
 
Call Girl Koramangala | 7001305949 At Low Cost Cash Payment Booking
Call Girl Koramangala | 7001305949 At Low Cost Cash Payment BookingCall Girl Koramangala | 7001305949 At Low Cost Cash Payment Booking
Call Girl Koramangala | 7001305949 At Low Cost Cash Payment Bookingnarwatsonia7
 
Glomerular Filtration and determinants of glomerular filtration .pptx
Glomerular Filtration and  determinants of glomerular filtration .pptxGlomerular Filtration and  determinants of glomerular filtration .pptx
Glomerular Filtration and determinants of glomerular filtration .pptxDr.Nusrat Tariq
 
Noida Sector 135 Call Girls ( 9873940964 ) Book Hot And Sexy Girls In A Few C...
Noida Sector 135 Call Girls ( 9873940964 ) Book Hot And Sexy Girls In A Few C...Noida Sector 135 Call Girls ( 9873940964 ) Book Hot And Sexy Girls In A Few C...
Noida Sector 135 Call Girls ( 9873940964 ) Book Hot And Sexy Girls In A Few C...rajnisinghkjn
 
Glomerular Filtration rate and its determinants.pptx
Glomerular Filtration rate and its determinants.pptxGlomerular Filtration rate and its determinants.pptx
Glomerular Filtration rate and its determinants.pptxDr.Nusrat Tariq
 
Call Girls Frazer Town Just Call 7001305949 Top Class Call Girl Service Avail...
Call Girls Frazer Town Just Call 7001305949 Top Class Call Girl Service Avail...Call Girls Frazer Town Just Call 7001305949 Top Class Call Girl Service Avail...
Call Girls Frazer Town Just Call 7001305949 Top Class Call Girl Service Avail...narwatsonia7
 
VIP Call Girls Lucknow Nandini 7001305949 Independent Escort Service Lucknow
VIP Call Girls Lucknow Nandini 7001305949 Independent Escort Service LucknowVIP Call Girls Lucknow Nandini 7001305949 Independent Escort Service Lucknow
VIP Call Girls Lucknow Nandini 7001305949 Independent Escort Service Lucknownarwatsonia7
 
See the 2,456 pharmacies on the National E-Pharmacy Platform
See the 2,456 pharmacies on the National E-Pharmacy PlatformSee the 2,456 pharmacies on the National E-Pharmacy Platform
See the 2,456 pharmacies on the National E-Pharmacy PlatformKweku Zurek
 
Kolkata Call Girls Services 9907093804 @24x7 High Class Babes Here Call Now
Kolkata Call Girls Services 9907093804 @24x7 High Class Babes Here Call NowKolkata Call Girls Services 9907093804 @24x7 High Class Babes Here Call Now
Kolkata Call Girls Services 9907093804 @24x7 High Class Babes Here Call NowNehru place Escorts
 
College Call Girls Vyasarpadi Whatsapp 7001305949 Independent Escort Service
College Call Girls Vyasarpadi Whatsapp 7001305949 Independent Escort ServiceCollege Call Girls Vyasarpadi Whatsapp 7001305949 Independent Escort Service
College Call Girls Vyasarpadi Whatsapp 7001305949 Independent Escort ServiceNehru place Escorts
 
VIP Call Girls Mumbai Arpita 9910780858 Independent Escort Service Mumbai
VIP Call Girls Mumbai Arpita 9910780858 Independent Escort Service MumbaiVIP Call Girls Mumbai Arpita 9910780858 Independent Escort Service Mumbai
VIP Call Girls Mumbai Arpita 9910780858 Independent Escort Service Mumbaisonalikaur4
 
High Profile Call Girls Kodigehalli - 7001305949 Escorts Service with Real Ph...
High Profile Call Girls Kodigehalli - 7001305949 Escorts Service with Real Ph...High Profile Call Girls Kodigehalli - 7001305949 Escorts Service with Real Ph...
High Profile Call Girls Kodigehalli - 7001305949 Escorts Service with Real Ph...narwatsonia7
 
call girls in green park DELHI 🔝 >༒9540349809 🔝 genuine Escort Service 🔝✔️✔️
call girls in green park  DELHI 🔝 >༒9540349809 🔝 genuine Escort Service 🔝✔️✔️call girls in green park  DELHI 🔝 >༒9540349809 🔝 genuine Escort Service 🔝✔️✔️
call girls in green park DELHI 🔝 >༒9540349809 🔝 genuine Escort Service 🔝✔️✔️saminamagar
 

Recently uploaded (20)

Call Girl Service Bidadi - For 7001305949 Cheap & Best with original Photos
Call Girl Service Bidadi - For 7001305949 Cheap & Best with original PhotosCall Girl Service Bidadi - For 7001305949 Cheap & Best with original Photos
Call Girl Service Bidadi - For 7001305949 Cheap & Best with original Photos
 
High Profile Call Girls Jaipur Vani 8445551418 Independent Escort Service Jaipur
High Profile Call Girls Jaipur Vani 8445551418 Independent Escort Service JaipurHigh Profile Call Girls Jaipur Vani 8445551418 Independent Escort Service Jaipur
High Profile Call Girls Jaipur Vani 8445551418 Independent Escort Service Jaipur
 
Russian Call Girl Brookfield - 7001305949 Escorts Service 50% Off with Cash O...
Russian Call Girl Brookfield - 7001305949 Escorts Service 50% Off with Cash O...Russian Call Girl Brookfield - 7001305949 Escorts Service 50% Off with Cash O...
Russian Call Girl Brookfield - 7001305949 Escorts Service 50% Off with Cash O...
 
Call Girls Electronic City Just Call 7001305949 Top Class Call Girl Service A...
Call Girls Electronic City Just Call 7001305949 Top Class Call Girl Service A...Call Girls Electronic City Just Call 7001305949 Top Class Call Girl Service A...
Call Girls Electronic City Just Call 7001305949 Top Class Call Girl Service A...
 
Call Girls Jayanagar Just Call 7001305949 Top Class Call Girl Service Available
Call Girls Jayanagar Just Call 7001305949 Top Class Call Girl Service AvailableCall Girls Jayanagar Just Call 7001305949 Top Class Call Girl Service Available
Call Girls Jayanagar Just Call 7001305949 Top Class Call Girl Service Available
 
97111 47426 Call Girls In Delhi MUNIRKAA
97111 47426 Call Girls In Delhi MUNIRKAA97111 47426 Call Girls In Delhi MUNIRKAA
97111 47426 Call Girls In Delhi MUNIRKAA
 
call girls in Connaught Place DELHI 🔝 >༒9540349809 🔝 genuine Escort Service ...
call girls in Connaught Place  DELHI 🔝 >༒9540349809 🔝 genuine Escort Service ...call girls in Connaught Place  DELHI 🔝 >༒9540349809 🔝 genuine Escort Service ...
call girls in Connaught Place DELHI 🔝 >༒9540349809 🔝 genuine Escort Service ...
 
Case Report Peripartum Cardiomyopathy.pptx
Case Report Peripartum Cardiomyopathy.pptxCase Report Peripartum Cardiomyopathy.pptx
Case Report Peripartum Cardiomyopathy.pptx
 
Call Girl Koramangala | 7001305949 At Low Cost Cash Payment Booking
Call Girl Koramangala | 7001305949 At Low Cost Cash Payment BookingCall Girl Koramangala | 7001305949 At Low Cost Cash Payment Booking
Call Girl Koramangala | 7001305949 At Low Cost Cash Payment Booking
 
Glomerular Filtration and determinants of glomerular filtration .pptx
Glomerular Filtration and  determinants of glomerular filtration .pptxGlomerular Filtration and  determinants of glomerular filtration .pptx
Glomerular Filtration and determinants of glomerular filtration .pptx
 
Noida Sector 135 Call Girls ( 9873940964 ) Book Hot And Sexy Girls In A Few C...
Noida Sector 135 Call Girls ( 9873940964 ) Book Hot And Sexy Girls In A Few C...Noida Sector 135 Call Girls ( 9873940964 ) Book Hot And Sexy Girls In A Few C...
Noida Sector 135 Call Girls ( 9873940964 ) Book Hot And Sexy Girls In A Few C...
 
Glomerular Filtration rate and its determinants.pptx
Glomerular Filtration rate and its determinants.pptxGlomerular Filtration rate and its determinants.pptx
Glomerular Filtration rate and its determinants.pptx
 
Call Girls Frazer Town Just Call 7001305949 Top Class Call Girl Service Avail...
Call Girls Frazer Town Just Call 7001305949 Top Class Call Girl Service Avail...Call Girls Frazer Town Just Call 7001305949 Top Class Call Girl Service Avail...
Call Girls Frazer Town Just Call 7001305949 Top Class Call Girl Service Avail...
 
VIP Call Girls Lucknow Nandini 7001305949 Independent Escort Service Lucknow
VIP Call Girls Lucknow Nandini 7001305949 Independent Escort Service LucknowVIP Call Girls Lucknow Nandini 7001305949 Independent Escort Service Lucknow
VIP Call Girls Lucknow Nandini 7001305949 Independent Escort Service Lucknow
 
See the 2,456 pharmacies on the National E-Pharmacy Platform
See the 2,456 pharmacies on the National E-Pharmacy PlatformSee the 2,456 pharmacies on the National E-Pharmacy Platform
See the 2,456 pharmacies on the National E-Pharmacy Platform
 
Kolkata Call Girls Services 9907093804 @24x7 High Class Babes Here Call Now
Kolkata Call Girls Services 9907093804 @24x7 High Class Babes Here Call NowKolkata Call Girls Services 9907093804 @24x7 High Class Babes Here Call Now
Kolkata Call Girls Services 9907093804 @24x7 High Class Babes Here Call Now
 
College Call Girls Vyasarpadi Whatsapp 7001305949 Independent Escort Service
College Call Girls Vyasarpadi Whatsapp 7001305949 Independent Escort ServiceCollege Call Girls Vyasarpadi Whatsapp 7001305949 Independent Escort Service
College Call Girls Vyasarpadi Whatsapp 7001305949 Independent Escort Service
 
VIP Call Girls Mumbai Arpita 9910780858 Independent Escort Service Mumbai
VIP Call Girls Mumbai Arpita 9910780858 Independent Escort Service MumbaiVIP Call Girls Mumbai Arpita 9910780858 Independent Escort Service Mumbai
VIP Call Girls Mumbai Arpita 9910780858 Independent Escort Service Mumbai
 
High Profile Call Girls Kodigehalli - 7001305949 Escorts Service with Real Ph...
High Profile Call Girls Kodigehalli - 7001305949 Escorts Service with Real Ph...High Profile Call Girls Kodigehalli - 7001305949 Escorts Service with Real Ph...
High Profile Call Girls Kodigehalli - 7001305949 Escorts Service with Real Ph...
 
call girls in green park DELHI 🔝 >༒9540349809 🔝 genuine Escort Service 🔝✔️✔️
call girls in green park  DELHI 🔝 >༒9540349809 🔝 genuine Escort Service 🔝✔️✔️call girls in green park  DELHI 🔝 >༒9540349809 🔝 genuine Escort Service 🔝✔️✔️
call girls in green park DELHI 🔝 >༒9540349809 🔝 genuine Escort Service 🔝✔️✔️
 

Communicable disease q&a

  • 1. Situation 1: Information dissemination is an integral element of health promotion, and disease prevention. The nurse is in the best position to do health education activities. 1. A nurse is providing instructions to a pregnant client with genital herpes about measures that need to be implemented to protect the fetus. The nurse tells the nurse that: a. Daily administration of acyclovir (Zovirax) is necessary during the entire pregnancy b. Total abstinence from sexual intercourse is necessary during the entire pregnancy c. Sitz baths need to be taken every 4 hours while awake if vaginal lesions are present d. A cesarean section will be necessary if vaginal lesions are present at the time of labor 2. While the nurse is counseling a group of mothers about sexually transmitted diseases, one mother asks the nurse which sexually transmitted disease is most detrimental to their baby's safety during pregnancy? The nurse should respond that it would be: a. Gonorrhea c. Genital herpes b. Syphilis d. Trichomonas vaginalis 3. During an infection control seminar, the speaker specified that prevention of the spread of HIV include the following measures EXCEPT: a. Patients with AIDS should be isolated b. Blood and other specimens should be labeled AIDS Precaution c. Needles should be disposed into a puncture resistant container d. Blood spills should be cleaned with chlorox 4. A nurse is giving health education to a client diagnosed with HIV. The nurse determines that the client does not need further teaching if the client states that the most effective method known to control the spread of HIV infection is: a. Premarital serologic screening b. Prophylactic treatment of exposed people c. Laboratory screening of pregnant women d. Sex education about preventive behaviors. 5. The following are preventions of gonorrhea transmission EXCEPT: a. Sex education b. Case finding c. Incidence to be reported to health authorities d. Administration of ophthalmic prophylaxis, as ordered Situation 2: The nursing process is applied in any health care setting. 6. A child is diagnosed with scarlet fever. The nurse assesses the child knowing that which of the following is not a clinical manifestation associated with this disease? a. Pastia's sign b. White strawberry tongue c. Edematous and beefy, red colored pharynx d. Koplik spots 7. A nurse provides instructions to the mother of a child with mumps regarding respiratory precautions. The mother asks the nurse the length of time required for the respiratory precautions. The nurse most appropriately responds that: a. “Respiratory precautions are necessary for the entire time of illness.” b. “Respiratory precautions are necessary until the swelling is gone.” c. “Respiratory precautions are indicated during the period of communicability.” d. “Respiratory precautions are indicated for 18 days after the onset of parotid swelling.” 8. A 6-month-old infant receives DPT immunization at the well-baby clinic. The mother returns home and calls the clinic to report that the infant has developed swelling and redness at the site of injection. The nurse tells the mother to: a. Apply a warm pack to the injection site b. Bring the infant back to the clinic c. Apply an ice pack to the injection site d. Monitor the infant for fever 9. A nurse is preparing the plan of care for a patient with herpes genitalis. What would be the priority nursing diagnosis for the patient?
  • 2. a. Disturbed Sleep pattern b. Imbalance in Nutrition: Less than Body Requirements c. Alteration in Comfort: Pain d. Ineffective Breathing Pattern 10. The nurse on duty, based on her assessment findings suspects the presence of diphtheria. Which of the following will confirm her suspicion? a. Pharyngotonsilar congestion b. Grayish membrane at hard palate c. Whitish plaque at buccal mucosa d. Increased lacrimation Situation 3: There are 170 Bilhariasis endemic municipalities. You are assigned in one of these municipalities. 11. In order to confirm the diagnosis of Snail fever, you advise patient to have, which of these examination: a. X-ray of the abdomen b. Urinalysis c. Stool examination d. CBC 12. You know that the mode of transmission of Katayama fever is: a. Contact with affected stray animals b. Contact with water infected with cercaria c. Infected flies and rodent d. Use of sanitary toilets 13. Which among the following is the drug of choice for Bilhariasis? a. Biltricide c. Chloramphenicol b. Hetrazan d. Tetracycline 14. The following are preventive measures for Schistosomiasis EXCEPT: a. Use of safe water b. Avoid bathing and washing in infested waters c. Use of sanitary toilets d. Elimination of breeding sites of mosquitoes 15. Which of the following is NOT a complication of Bilhariasis? a. Liver cirrhosis and portal hypertension b. Cor pulmonale, pulmonary hypertension c. Meningitis and hepatomegally d. Ascitis and renal failure Situation 4: Endemic malaria occurs in the topic and subtopic areas where socioeconomic condition is very poor. 16. During your assessment, which among the following signs and symptoms need referral to a secondary or tertiary facility? a. Sweating and headache b. Icterus and shock c. Fever and chills d. Renal or liver failure 17. Laboratory confirmation of malaria is done on a blood film. What do you expect to see in the film? a. Antibodies c. Malarial purinates b. Malarial parasites d. Antigen 18. The mode of transmission of this disease is through the bite of an infected female mosquito called: a. Aminophylline c. Anopheles b. Aedes poecillus d. Aedes egypti 19. Which of the following is NOT an anti-malarial drug? a. Sulfadoxine c. Tetracycline b. Amoxicillin d. Quinidine
  • 3. 20. As a preventive measure for malarais, you educate people living in malaria endemic areas to do which of the following? A. Avoid going out between 12nn to 3am B. Take chloroquine tablets once a day C. Apply insect repellant on house walls D. Use long sleeved shirts when going out at night E. Plant neem tree in their backyards F. Clear hanging branches nears rivers a. A, B, C, D, E, F c. B, C, D, E, F b. C, D, E, F d. B, C, D, E Situation 5: A nurse is having her duty in a public health clinic. She encounters multiple cases of sexually transmitted diseases. 21. A nurse is collecting data from a prenatal client. The nurse determines that which of the following places the client into the high risk category for contracting human immunodeficiency virus? a. Living in an area where HIV infections are minimal b. A history of IV drug use in the past year c. A history of one sexual partner within the past 10 years d. A spouse who is heterosexual and had only 1 sexual partner in the past 10 years 22. Which among the following informations is true of HIV? a. It can be transmitted via body fluids such as blood, semen, urine, and perspiration. b. Blood, semen, and breast milk have higher concentrations of HIV than urine, saliva, vomitus and stool c. A client who by history may be exposed to HIV but test negative for HIV antibodies can no longer infect others d. Enzyme-linked immunosorbent assay (ELISA) is the confirmatory test for HIV 23. Hepatitis B infection is established by the presence of hepatitis B antigen-antibody systems in the blood. Which of the following is NOT true? a. Presence of HBsAG is the serological marker to establish the diagnosis of hepatitis B. b. If the serological marker is present in the blood after 3 months, it indicates a carrier state or chronic hepatitis. c. The presence of anti-HBS indicates recovery and immunity to hepatitis B. d. Presence of HbeAG determines the infective state of the client 24. The nurse is talking to a young female client in the health clinic who is concerned she may have sexually transmitted disease. The nurse commends her for seeking medical care. The nurse explains that the major reason treatment of majority of STDs is delayed because: a. Client is embarrassed b. Symptoms are though to be caused by something else c. Symptoms are ignored d. The client does not manifest signs and symptoms 25. The nurse is very active in contact tracing and epidemiologic treatment of all gonococcal identified contacts to prevent: a. Development of resistant strains b. Asymptomatic infections c. Non-venereal transmission d. Reinfection Situation 6: Avian Influenza is an infectious disease of birds that can cause serious illness in humans. 26. The importation of chicken from countries with outbreak of avian flu is banned by which law? a. RA 280 c. PD 280 b. EO 280 d. AO 280 27. The major causes of death in avian flu are the following except: a. Severe viral pneumonia b. Respiratory distress c. Multiple organ failure d. Dehydration from severe vomiting and diarrhea
  • 4. 28. A suspected case of avian flu would be identified if a person exhibits which of the following manifestations? a. Body weakness, fever, vomiting, diarrhea, cough, and anorexia b. Fever, body weakness cough, dyspnea, and sore throat c. Fever, cough, sore throat, diarrhea, bloody stool, and hematemesis d. Hemoptysis, difficulty breathing, sore eyes, vomiting and diarrhea 29. It is necessary to institute which of the following measures to birds suspected of being exposed to the virus? a. Vaccination of poultry b. Killing of all infected or exposed birds c. Mixing antiviral to feeds and water supplied to poultry suspected of being exposed to avian flu d. All of the above 30. Incubation period for avian influenza is: a. 10 days c. 21 days b. 3 days d. 7 days answers------------------------------------------------------------- 1. D. In mothers with genital herpes, the fetus can be contaminated after membranes rupture or with vaginal delivery. Therefore, for women with active lesions, either recurrent or primary at the time of labor, delivery should be by cesarean to prevent the fetus from coming into contact with the genital herpes. A. The safety of acyclovir had not been established during pregnancy and should only be used when life-threatening infection is present. C. Abstinence should be advised while the lesions are present, until they become culture- negative. D. This promotes healing. 2. A. Syphilis may cross the placenta, causing congenital syphilis. It usually leads to spontaneous abortion, and it increases the incidence of mental subnormality and physical deformities in fetus. B. In gonorrhea, the fetus is contaminated at the time of delivery. It places the neonate at risk for ophthalmia neonatorum, pneumonia and sepsis, but these can be avoided via cesarean delivery. C. In genital herpes, fetus is contaminated after membranes rupture or with vaginal delivery. Mother should deliver via cesarean section. 3. A. Patients with AIDS are immunocompromised. Reverse isolation is required. But this does no prevent the spread of AIDS. B, C and D are universal preventive measures for AIDS. 4. D 5. D. This does not prevent transmission of gonorrhea. It prevents the occurrence of ophthalmia neonatorum. 6. D. Koplik spots are associated with rubeola. A. Pastia's sign describes a rash that is seen in scarlet fever that will blanch with pressure except in areas of deep creases and folds of the joints. B The tongue is initially coated with a white furry covering with red projecting papillae (white strawberry tongue). C. The pharynx is edematous and beefy red in color.
  • 5. 7. A. Mumps is transmitted via direct contact or droplet spread from an infected person and possibly by contact with the urine. Respiratory precautions are indicated during the period of communicability. 8. C. Occasionally, tenderness, redness, or swelling may occur at the site of the injection. This can be relieved with ice packs for the first 24 hours followed by warm compresses if the inflammation persists. 9. C. Genital herpes is characterized by painful vesicles affecting the external genitalia, vagina and cervix. 10. B. The presence of a grayish membrane on the hard palate, or pseudomembrane is a pathognomonic sign of diphtheria. Whitish plaque at buccal mucosa, or koplik spots, are seen in a rubeola infection (measles). 11. C. The male and female parasites live in blood vessels of intestines and liver, but the eggs are laid in the terminal capillary vessels in the submucosa of the intestines, and through the ulcerations reach the lumen of the intestines and pass out with the feces. 12. B. Infection occurs when skin comes in contact with contaminated fresh water where cercariae (free swimming larval forms of schistosomes) are living. 13. A. Praziquantel (Biltricide) is the drug of choice for Schistosomiasis. B. Hetrazan is the drug of choice for Filiariasis C. Chloramphenicol is the drug of choice for Typhoid fever D. Tetracycline is the drug of choice for Cholera 14. D. This is a preventive measure for vector-borne diseases. Schistosomiasis is a water-borne disease. A. Drinking water should be free of cercariae. B. Schistosomiasis can be transmitted through exposure to contaminated water. C. The use of sanitary toilets prevents viable eggs from reaching bodies of water containing intermediate snail host. 15. C. Meningitis is not a complication of Bilhariasis. 16. B. Malarial parasites invade and destroy red blood cells. 17. B. As sporozoites from the salivary gland of a female Anopheles mosquito are injected under the skin, they travel into the bloodstream to the liver and mature within hepatocytes. Up to 30,000 parasites are then released into the bloodstream as merozoites and produce a symptomatic infection as they invade and destroy red blood cells.
  • 6. 18. C. B. This is the vector for Filariasis. D. This is the vector for Dengue. 19. B. A. Sulfadoxine (Sulfalene) 50 mg C. Tetracycline hydrochloride 250mg/capsule D. Quinidine sulfate 300 mg table; Quinidine hydrochloride 300 mg/mL, 2 ML ampule 20. B. A and b are not included. The vector's peak biting hours are 9pm to 3am. Chloroquine is taken at weekly intervals. 21. B. Women who fall into the high-risk category for HIV infection include those with persistent and recurrent sexually transmitted diseases or a history of multiple sexual partners, and those who use or have used IV drugs. A, C and D do not contribute to the incidence of contracting HIV. 22. B. This is true of HIV. A. Perspiration does not contain the virus. C. This is not always true for HIV. Clients may be infected with the virus, but has not yet produced antibodies, thereby testing negative, but being capable of infecting others. It usually takes 6 to 12 weeks (other books: 1 to 3 months) for a host to manufacture detectable HIV antibodies. D. Western blot is the confirmatory test for HIV. 23. B. Presence of hepatitis B surface antigens (HBsAG) in the blood after 6 months indicates a carrier state or chronic hepatitis. A, C and D are true.Hepatitis B early antigen is detected in the blood about 1 week after the appearance of HBsAG. Its presence determines the infective state of the clien 24. A 25. B. 80% of gonoccocal infection particularly in females have no symptoms. 26. B 27. D. Avian flu is not associated with vomiting and diarrhea.
  • 7. 28. B A and D. Vomiting and diarrhea are not associated with bird flu. C. Diarrhea, bloody stool, and hematemesis not associated with bird flu. 29. B. One control measure for Avian flu is the rapid destruction (culling or stamping out) of all infected or exposed birds. Other control measures include proper disposal of carcasses; quarantining; rigorous disinfection of farms; and restriction on the movement of live poultry. 30. B. The incubation period ranges 2-4 days. Questions-------------------------------------- Situation 13. Leprosy is an ancient disease and is a leading cause of permanent physical disability among the communicable diseases. It is a chronic mildly communicable disease that mainly affects the skin, the peripheral nerves, the eyes and mucosa of the upper respiratory tract. 61. Assessment is done upon seeing a patient with leprosy. The following are early signs and symptoms, except: 1. Appearance of a skin patch 2. Loss of sensation on skin lesion 3. Inability to close eyelids 4. Hair growth over the lesion 5. Painful nerves 6. Loss of eyebrow 7. Ulcers that do not heal 8. Sinking of the nose bridge a. 3, 5 and 7 c. 3, 6 and 8 b. 2, 5 and 8 d. 2, 6 and 7 62. A person can get leprosy through the following except: 1. Inhalation of the infectious organism through a broken skin 2. Repeated skin to skin contact with untreated individual 3. Inhalation of the infectious organism from the nasal discharges of an untreated individual 4. Contact with articles soiled with discharges from lesions of leprosy patient a. 1 only c. 4 only b. 1 and 3 d. 3 and 4 63. Philip is a 55 year-old resident who is affected by leprosy and is on schedule for Multidrug Therapy. Which daily treatment is usually prescribed by a physician for lepromatous and borderline type in 28 days? a. Dapsone 10 mg and Rifampicin 600 mg b. Clofazimine 50 mg and Rifampicin 50 mg c. Rifampicin 150 mg and Dapsone 50 mg d. Clofazimine 50 mg and Dapsone 100 mg 64. Only in rare instances is there really a need to use laboratory and other investigations to confirm a diagnosis. However, when a clinical diagnosis is doubtful. This optional procedure is done to prevent misclassification and wrong treatment. a. Direct Sputum Smear Microscopy (DSSM) b. Slit Skin Smear (SSS) examination c. Mass Blood Smear (MBS) collection d. Purified Purine Derivative (PPD) test 65. Preventive aspect of care is a priority consideration. Which of the following vaccine is also preventive measure? a. BCG c. Gammaglobulin b. DPT d. ATS Situation 14. Nurse Lorie is taking care of Anne, 15 months old, who is admitted to the hospital with Rubeola. She has conjunctivitis, coryza, cough, rashes and temperature of 38ºC.
  • 8. 66. Which of the following statements is true of Rubeola? 1. It is caused by a filterable virus 2. The prodrome period lasts for about 2 weeks 3. The rashes doesn't end in a branny desquamation 4. It is an acute highly communicable infection 5. The rash usually begins behind the ears and neck 6. The cause of death is usually from complications 7. Serum antibodies appear on the third day of the incubation period 8. A morbilliform rash appears on the 3rd or 4th day a. 1, 2, 3, 5 and 6 c. 2, 3, 4, 5 and 7 b. 3, 4, 5, 7 and 8 d. 1, 4, 5, 6 and 8 67. Nurse Lorie knows that the incubation period of Rubeola is: a. 10 days from exposure to appearance of fever and about 14 days until rash appears b. 2-3 weeks, commonly 13 to 17 days c. 12 to 26 days, usually 18 days d. Varies from 3 days to 1 month or more, falling between 7-14days in high proportion of cases 68. Anne's immunity from her mother lasted only 6 months. She was brought by her mother because of spots in her buccal mucosa. What do you call these spots? a. Pseudomembrane c. Koplik spots b. Rose spots d. Forscheimer's spots 69. The characteristic signs of measles are which of the following group? a. Skin eruptions which are abundant on covered areas of the body than on the exposed areas b. Rashes which spread from the face to the trunk and limbs, conjunctivitis, high fever and tiny white spots in the mucosa inside the cheek c. Vomiting, headache, fine petechial and morbilliform rashes and epistaxis d. Severe backache and rashes which are more abundant on extremities than on the trunk 70. A nursing intervention that is important in caring for Anne would include the following, except: a. Administer as prescribed antipyretics to reduce fever b. Ensuring that the room is well lighted c. Wearing face mask when administering nursing care to the child d. Increase fluid intake of the child by drinking water frequently Situation 15. Vangie is a 15 year-old child who is spending Christmas Vacation in Pangasinan. A week before the vacation ends, she developed fever, diarrhea and rose spots on her abdomen. 71. Based on Vangie's clinical findings, Nurse Ulysses will suspect Vangie to have contacted which infection? a. Dysentery c. Cholera b. Amoebiasis d. Typhoid fever 72. Vangie's condition is caused by a: a. Virus c. Helminth b. Bacteria d. Protozoa 73. Nurse Ulysses provided the family with health education to prevent transmission of infection as the disease can be transmitted to family members through the following, except: a. Flies b. Contaminated food and water c. Airborne transmission d. Direct contact 74. The characteristic signs of the said condition in #71 are which of the following? 1. Intermittent fever 2. Continued fever 3. Loss of appetite 4. Slow pulse 5. Rose spots on trunk and abdomen 6. Diarrhea 7. Rice watery stool 8. Bloody stool
  • 9. a. 2, 3, 4, 5 and 6 c. 1, 3, 4, 5 and 7 b. 1, 3, 4, 5 and 6 d. 2, 3, 4, 5 and 8 75. As a preventive control measure for the said condition in #71, which of the following must be discussed in the health education program for the pubic? 1. Sanitary disposal of human feces and maintenance of fly proof latrine 2. Proper food handling and preparation 3. Removal of stagnant water in empty water drums and flower pots 4. Use of repellants and insecticides a. 1 and 4 c. 3 and 4 b. 2 and 3 d. 1 and 2 Situation 16. According to a nationwide survey on soil-transmitted helminthiasis (STH) among children aged 12-71 months old done by the DOH, UP and UNICEF in 2003, it showed a cumulative prevalence rate of 66 percent. Studies have shown that STH infected children are prone to acquire other infections. There is a need to sustain mass deworming for 3 consecutive years to immediately control intestinal parasitism and other parasites among these infected children at the critical time of their growth and development. 76. Antihelminthic drugs used include: a. Albendazole and Mebendazole b. Metronidazole and Albendazole c. Cotrimoxazole and Metronidazole d. All of the above 77. It is a parasitic disease that has similar manifestation with Pulmonary Tuberculosis. What is it? a. Trichuriasis c. Ascariasis b. Filariasis d. Paragonimiasis 78. The treatment of choice for the condition in #77 is: a. Bithionol c. Praziquantel b. Ethambutol d. Metronidazole 79. The three major causes of intestinal parasitic infections in the Philippines are the following, except: a. Trichuriasis c. Ancylostomiasis b. Taeniasis d. Ascariasis 80. This parasite enters the intestinal mucosa and could be spread through autoinfection. This is typically exemplify by: a. hookworm c. whipworm b. tapeworm d. pinworm Situation 17. Lord Voldemort complained of dizziness, vomiting, headache and numbness of the face especially around the mouth after 45 minutes of eating shellfish. He was rushed to the hospital and was diagnosed with Paralytic Shellfish Poisoning (PCP) or Red Tide Poisoning. 81. In addition to the above mentioned signs and symptoms manifested by Lord Voldemort, he may also manifest the following: a. Blurred vision and rapid rise in the blood pressure b. Seizures, bradycardia, hematemesis and chest pain c. Tachycardia, dysphagia, paresthesia and ataxia d. Floating sensation, hypertension, bradycardia and abdominal pain 82. Which of the following statement is not true about red tide poisoning? a. The toxin is totally destroyed upon cooking b. Symptoms occur within minutes or several hours after ingestion of poisonous shellfish c. It is caused by a plankton d. Symptoms include tingling sensation and paresthesia 83. Nurse Bellatrix is giving health education to the family of Lord Voldemort regarding Red Tide Poisoning. Lord Voldemort asked the nurse, “Does cooking the shellfish make it safe to eat?” Nurse Bellatrix's best response is: a. “Yes, as long as you cook the shellfish properly at a boiling point.” b. “No, the poisons are not destroyed by cooking.” c. “Yes, the toxins are easily destroyed by heat.”
  • 10. d. “No, the toxins are destroyed by freezing.” 84. The mother of Lord Voldemort asked Nurse Bellatrix, “Which sea foods can transmit red tide poisoning to human?” The nurse response included the following sea foods, except: a. crab meat c. clams and mussels b. oysters and scallops d. sea cucumber 85. Management of Red Tide Poisoning includes: a. Do not induce vomiting b. Giving of medications c. Give coconut milk and sodium bicarbonate d. The toxin is totally destroyed upon cooking Situation 18. Severus was admitted with manifestation of jaundice accompanied by pruritus and urticaria. The patient was diagnosed with Hepatitis A. 86. Hepatitis A is also known by the following names, except: a. Infectious hepatitis c. Epidemic hepatitis b. Catarrhal jaundice d. Inoculation hepatitis 87. Severus asked the nurse, “Why don't you give me some medication to help me get rid of this problem?” Nurse Neville's best response would be: a. “Sedatives can be given to help you relax.” b. “We can give you immune serum globulin.” c. “There are no specific drugs used to treat hepatitis.” d. “Vitamin supplements are frequently helpful and hasten recovery.” 88. Nurse Neville has instructed Severus about the type of diet should be eaten. The lunch selection that would indicate the client's understanding and compliance with the dietary principles taught is: a. Turkey salad, French fries and sherbet b. Salad, sliced chicken sandwich and gelatin dessert c. Cheeseburger, taco chips and chocolate pudding d. Cottage cheese, peanut butter sandwich and milk shake 89. The cooked food most likely to remain contaminated by the virus that causes Hepatitis A is which of the following? a. Canned tuna c. Fried tuna belly b. Broiled shrimp d. Steamed lobster 90. Which of the following signs and symptoms occur in a patient with Hepatitis A? 1. Loss of appetite 2. Enlargement of the lymph nodes 3. Tachycardia 4. Jaundice 5. Bradycardia 6. Urticaria 7. Clay-colored stools a. 1, 2, 4, 6 and 7 c. All except 5 b. 2, 3, 4, and 6 d. All of the above Situation 19. During the rainy season, several areas in Davao City becomes flooded with water. Leptospirosis is one infection that is common during this time of year. 91. Leptospirosis is also known with the following names, except: a. Mud fever and Flood fever b. Weil's disease and Trench fever c. Catarrhal jaundice and Ragpicker disease d. Spiroketal jaundice and Japanese Seven Days Fever 92. Dobby, who is suspected of having Leptospirosis, should have his urine be tested for the presence of the microorganism: a. During the febrile period b. During the first week of the illness
  • 11. c. During the first six months after contracting the infection d. After the 10th day of illness 93. Leptospirosis can be transmitted through: a. Open skin contact c. Droplets b. Airborne d. Sexually transmitted 94. Albus asked if it is possible to be infected while swimming in public pools or rivers. Nurse Hermione replied that: a. “This is not possible as long as you have no open wounds while swimming in public pools.” b. “Infection with leptospirosis is possible with recreational swimming in water contaminated with urine of human and animals having the infection.” c. “You can protect yourself by applying 70% alcohol before swimming d. “There is no danger during day time because leptospires die when exposed to sunlight. 95. Management and treatment of Leptospirosis is symptomatic but a drug medication is prescribed at 200 mg orally once a week for prevention. What is the drug of choice? a. Chloramphenicol c. Zidovudine b. Doxycycline d. Streptomycin Situation 20. Rabies remains a public health problem in the Philippines. Approximately 300 to 600 Filipinos die of rabies every year. The Philippines has one of the highest prevalence rates of rabies in the whole world. 96. Dolores Umbridge, while on her way home from work, was bitten by a stray dog. Nurse Sirius asked Dolores what she did immediately after the incident. Dolores's response indicates correct understanding of first aid measure when she says that: a. I went to the hospital and reported the incident b. I ran after the dog and had bitten it too c. I washed my wound immediately with soap and water d. I went to the drug store and bought 500 mg antibiotics 97. Nurse Sirius interviewed Dolores and her mother at the Emergency Room to take the history before seeing a doctor. The nurse told Dolores's mother not to kill the dog because the dog will be: a. Injected with a vaccine b. Confined at a veterinary clinic c. Observed for 14 days d. Treated with antibiotics against rabies 98. It is the objective of the Department of Health (DOH) to reduce the incidence of Human Rabies from 7 per million to 1 per million population by: a. 201 b. 2015 c. 2020 d. 2025 99. Considering that rabies is one of the most acutely fatal infections which causes the death of approximately 300 to 600 Filipinos annually, voluntary pre-exposure prophylaxis among people who are at risk is highly recommended. These people include: a. Barangay tanod officials patrolling the streets at night, side walk vendors, veterinarians and animal handlers b. All people living within an area where there are dogs that roam their neighborhood c. Pet owners, animal handlers, health personnel working in anti-rabies units and children below 15 years old d. Pregnant women, side walk vendors, pet owners, veterinarians and infants 100. What is the most common caused of death in patients with Rabies? a. Seizures c. Hemorrhage b. Respiratory Paralysis d. Hydrophobia answers---------------------------------------------------------------- 61. Answer: C Rationale: The following are late signs and symptoms of Leprosy: (3) Inability to close eyelids (lagophthalmos), clawaing of the fingers and toes, contractures, (6) loss of eyebrow (madarosis), enlargement of the breast in males (gynecomastia), chronic ulcers and (8) sinking of the nose bridge.
  • 12. 62. Answer: A Rationale: The mode of transmission of leprosy are the following: Airborne (inhalation of droplet/spray from coughing and sneezing of untreated leprosy patient) and Prolonged skin-to-skin contact. 63. Answer: D Rationale: Multidrug Therapy is the method of treatment for leprosy. The daily treatment prescribed for a Multibacillary (lepromatous and borderline) type in 28 days is Clofazimine 50 mg and Dapsone 100 mg. 64. Answer: B Rationale: Slit Skin Smear (SSS) examination is an optional procedure done only when a clinical diagnosis is doubtful. This prevent misclassification and wrong treatment. Options A and D is used for diagnosing Tuberculosis. Option C is used in Malaria. 65. Answer: A Rationale: Bacille Calmette-Guérin (BCG) vaccine is used as preventive measure for Tuberculosis. It can also prevent the occurrence of Leprosy since the two disease both came from the Mycobacterium family. 66. Answer: D Rationale: Rubeola or Measles is caused by a filterable virus (1). It is an acute highly communicable infection (4) characterized by fever, rashes and symptoms referable to upper respiratory tract; the eruption is preceded by about 2 days of coryza, during which stage grayish pecks (Koplik spots) may be found on the inner surface of the cheeks. A morbilliform rash appears on the 3rd or 4th day (8) affecting the face - usually beginning behind the ears and neck (5), body and extremities ending in branny desquamation. Death is due to complication (6) such as secondary pneumonia, usually in children under 2 years old 67. Answer: A Rationale: The incubation period of Rubeola is 10 days from exposure to appearance of fever and about 14 days until rash appears. Option B is for Chicken pox. Option C is for Mumps. Option D is for Tetanus. 68. Answer: C Rationale: Koplik spots are tiny grayish to whitish spots found in the buccal mucosa of a client with Rubeola. Option A is the pathognomonic sign of Diphtheria. Option B is found in clients with Typhoid fever. Option D is found in clients with Rubella 69. Answer: B Rationale: The characteristic signs of measles are which of the following: Rashes which spread from the face to the trunk and limbs ending in branny desquamation, conjunctivitis (Stimson's sign), high fever and tiny white spots in the mucosa inside the cheek 70. Answer: B Rationale: Clients with Rubeola has conjunctivitis (Stimson's sign), as a result an important nursing intervention is to protect the eyes of the client from glare of strong light as they are apt to be inflamed. Place the client in a dim-lighted room, not well lighted. 71. Answer: D Rationale: Typhoid fever is characterized by continued fever, anorexia (loss of appetite), slow pulse, involvement of lymphoid tissues, especially ulceration of Peyer's patches, enlargement of spleen, rose spots on trunk and abdomen and diarrhea. 72. Answer: B Rationale: Typhoid fever is caused by Salmonella typhosa or typhoid bacillus, which is a bacteria. 73. Answer: C Rationale: Typhoid fever can be transmitted through the following mode of transmission: direct or indirect contact with patient or carrier; contaminated food and water with flies as vectors; and improper food handling. It is not transmitted via airborne. 74. Answer: A Rationale: Typhoid fever is characterized by continued fever, anorexia (loss of appetite), slow pulse, involvement of lymphoid tissues, especially ulceration of Peyer's patches, enlargement of spleen, rose spots on trunk and abdomen and diarrhea. (7) Rice watery stools are seen in patients with Cholera. 75. Answer: D Rationale: Preventive control measures include sanitary disposal of human feces and maintenance of fly proof latrine and proper food handling and preparation. 76. Answer: A Rationale: Antihelminthic drugs used in treating parasitic infections include: Albendazole and Mebendazole (Vermox) that inhibits glucose and other nutrient uptake of helminth; Pyrantel embonate (Antiminth) that paralyzes intestinal tract of worm; and Thiabendazole (Mintezol) that interferes with parasitic metabolism. Metronidazole (Flagyl) is an anti-protozoan drug (options B and C). Cotrimoxazole (Bactrim) is an antibiotics (option C). 77. Answer: D Rationale: Commonly patients with Paragonimiasis are misdiagnosed to have Pulmonary Tuberculosis (PTB) and are treated as Tuberculosis patients. In fact, a study by Belizario et.al revealed that 56% of his subjects were non-responsive to multi-drug therapy for PTB but were positive for Paragonimiasis.
  • 13. 78. Answer: C Rationale: The treatment of choice for Paragonimiasis is Praziquantel (Biltrizide) 25 mg/kg body weight three times daily for three days. It is suitable for treatment of adults and children over four years of age, higher dose is needed for ectopic paragonimiasis. Bithionol (BITIN) is only an alternative drug for paragonimiasis (option A). Ethambutol (option B) is one of the multi-drug therapy for PTB. Metronidazole (option D) is an anti-protozoan drug. 79. Answer: B Rationale: The three major causes of intestinal parasitic infections in the Philippines are the following: (A) Trichuriasis or Whipworm (Trichuris trichiura); (C) Ancylostomiasis or Hookworm (Ancylostoma duodenale and Necator americanus); and (D) Ascariasis (Ascaris lumbricoides). 80. Answer: D Rationale: Pinworms (enterobiasis), which is caused by the parasite Enterobius vermicularis, is the most common helminthic infestation among children. It enters the intestinal mucosa and could be spread through autoinfection (when the child places fingers, used in scratching the anal part and containing the pinworm eggs, into the mouth). It is noted that pinworms caused severe itching as they lay their eggs. 81. Answer: c Rationale: The signs and symptoms of Red Tide Poisoning includes: numbness of the face especially around the mouth; vomiting; dizziness; headache; tingling sensation, paresthesia and eventual paralysis of hands and feet; Floating sensation and weakness; rapid pulse (tachycardia); difficulty of speech (ataxia); difficulty of swallowing (dysphagia); and total muscle paralysis with respiratory arrest and death occur in severe cases. 82. Answer: A Rationale: Paralytic Shellfish Poisoning (PCP) or Red Tide Poisoning is a syndrome of characteristic symptoms predominantly neurologic which occur within minutes or several hours after ingestion of poisonous shellfish (option B). It is caused by a single celled organism called dinoflagellates, which is commonly referred to as plankton (option C). One of its symptoms include tingling sensation or paresthesia (option D). Toxins in red tide is not totally destroyed upon cooking. 83. Answer: B Rationale: Toxins in red tide is not totally destroyed upon cooking hence consumers must be educated to avoid bi-valve mollusks like tahong, talaba, halaan, kabiya, abaniko (sun and moon shell or Asian scallop) when the red tide warning has been issued by the proper authorities. 84. Answer: D Rationale: Bi-valve mollusks such as oysters, clams, mussels, sun and moon shell or Asian scallop as well as crabs are avoided when the red tide warning has been issued by the proper authorities. There's no reported cases of red tide poisoning when eating sea cucumber. 85. Answer: C Rationale: Management of Red Tide Poisoning includes the following: induce vomiting; no definite medication indicated; drinking pure coconut milk, which weakens the toxic effect of red tide, and sodium bicarbonate solution (25 grams in ½ glass of water) may be taken. Drinking coconut milk and sodium bicarbonate solution is advised during the early stages of poisoning only. If given during the late stag, they may make the condition of the patient worse; Shellfish affected by red tide must not be cooked with vinegar as the toxin of Pyromidium increases (15 time greater) when mixed with acid; and do not eat even cooked shellfish because toxins of red tide are not totally destroyed upon cooking. 86. Answer: D Rationale: The other names of Hepatitis A are the following: Infectious hepatitis; Catarrhal jaundice; and Epidemic hepatitis. In option D, Inoculation hepatitis is the other name for Hepatitis B. 87. Answer: C Rationale: There are no specific drugs for the treatment of Hepatitis A. There is, however, a prophylaxis, which is an IM injection of gamma globulin. 88. Answer: B Rationale: The dietary requirement for a client with Hepatitis A is a low fat diet but high in sugar. Options A, C and D are incorrect because all or part of its choices are high in fats. Low fat diet include only lean meats, fish and poultry. The allowed vegetable oils may be used in preparing meats, fish and poultry; used in salad dressings; or in baked products. 89. Answer: D Rationale: Hepatitis A virus can be killed by cooking the food to boiling point, steaming the food will not kill the virus. 90. Answer: A Rationale: Hepatitis A has the following signs and symptoms: influenza-like such as headache; malaise and easy fatigability; loss of appetite or anorexia (1); abdominal discomfort/pain; nausea and vomiting; fever; enlargement of the lymph node or lymphadenopathy (2); jaundice (4) accompanied by pruritus and urticaria (6); and bilirubinemia with clay-colored stools (7). 91. Answer: C
  • 14. Rationale: Leptospirosis is also known with the following names: Mud fever and Flood fever (option A); Weil's disease and Trench fever (option B); and Spiroketal jaundice and Japanese Seven Days Fever (option D). In option C, Catarrhal jaundice is the other name for Hepatitis A while Ragpicker disease is the other name for Anthrax. 92. Answer: D Rationale: Leptospirosis can be diagnosed by its clinical manifestations, culture of the organism, examination of blood and CSF during the first week of illness and urine after the 10th day. 93. Answer: A Rationale: Leptospirosis can be transmitted through contact of the skin, especially open wounds with water, moist soil or vegetation contaminated with urine of infected host. 94. Answer: B Rationale: Infection with leptospirosis is possible with recreational swimming in water contaminated with urine of human and animals having the infection. 95. Answer: B Rationale: Doxycycline is the drug of choice for Leptospirosis. It is prescribed at 200 mg orally once a week for prevention 96. Answer: C Rationale: The wound must be immediately and thoroughly washed with soap and water. Antiseptics such as povidone iodine or alcohol may be applied. 97. Answer: C Rationale: Patients should consult a veterinarian or trained personnel to observe the pet for 14 days for signs of rabies. 98. Answer: A Rationale: The general objective of the Department of Health (DOH) is to reduce the incidence of Human Rabies from 7 per million to 1 per million population by 2010 and eliminate human rabies by 2015. 99. Answer: C Rationale: People who are at high risk of acquiring rabies includes the following: Pet owners; animal handlers; health personnel working in anti-rabies units; and children below 15 years old. 100. Answer: B Rationale: Without medical intervention, the rabies victim would usually last only for 2 to 6 days. Death is often due to respiratory paralysis. Question------------------------------------- Situation 7: Ralph, a student nurse began to feel joint pains and eye pains associated with high fever. He noticed some pinpoint rashes on his legs. He is diagnosed with Dengue fever. 31. In the prevention of dengue fever, which of the following measures is included? a. Mass vaccination during rainy season b. Prophylactic drug treatment with antivirals c. Removal of small collections of water such as flower vases d. Fogging or daily spraying of insecticides 32. The most effective control measure for dengue hemorrhagic fever is: a. Isolation of patient b. Mosquito control c. Health education d. Concurrent disinfection 33. The treatment and control of dengue hemorrhagic fever include all of the following EXCEPT: a. Treat shock with IVF b. Encourage vaccination c. Eradication of vector d. Community participation 34. The best nursing intervention in the management of dengue fever among patients admitted in the hospital would be:
  • 15. a. Observation b. Contact Isolation c. Administration of antivirals d. Platelet administration 35. Which is an independent nursing intervention for a patient with Dengue Hemorrhagic Fever? a. Give aspirin for fever and muscle pains b. Put ice bag over forehead if patient has nose bleed c. Place patient in prone position to relieve abdominal pain d. Give high fiber diet Situation 8: The BHW of Barangay Jacinto reported series of adults having cases of spasmodic coughing and fever for the past two weeks. The CHN in the area alarmed by the reports decided to investigate the community. 36. Which among the following diagnostic tests does NOT indicate active tuberculosis? a. Sputum smear positive times three b. Chest x-ray as diagnosed by the TBDC c. An induration of 10 post tuberculin test d. Chest x-ray after having three negative sputum smears 37. Mang Jose, 42 years old, went to the Chest Center to avail of the free Direct Sputum Smear Microscopy. Which among the following is a contraindication for sputum collection? a. Pleural effusion c. Hemoptysis b. TB meningitis d. Miliary TB 38. Mang Jose began his treatment regimen for tuberculosis. He complains of frequent tingling sensation of his feet. Which among the following diets shout be encouraged? a. Fish c. Milk b. Kangkong d. Beef 39. As a nurse working in the Barangay Health Center, it is essential to counsel the client on the side effects of the treatment regimen. Which among the following side effects should be reported immediately to the physician? a. Hemoptysis c. Flu-like symptoms b. Red-colored urine d. Yellowish palms 40. Which among the following is considered as the most important health teaching given to clients with Tuberculosis? a. Strict compliance to treatment regimen b. Frequent handwashing c. Proper disposal of soiled tissues d. Covering of mouth while coughing or sneezing Situation 9: The nurse is aware of the misuse of antibiotics , which resulted to the emergence of new strains of microorganisms that proves to effect a catastophic effect on human health. 41. The period between the first exposure and multiplication of the SRAS causative agent would be: a. 2-10 days c. 1-20 days b. 5-11 days d. 1-5 days 42. SARS remains to be an enigma among health practitioners and had proven its virulence. The nurse knows that the virus could be categorized as: a. Rhabdovirus c. Coronavirus b. Togavirus d. Paramyxovirus 43. Which of the following best defines standards precaution? a. These are guidelines created to prevent transmission of microorganisms in hospitals b. It is a strategy of assuming that all patients are infectious, and using barrier protections during nurse-client interactions c. These are precautions based on contagious or epidemiologically significant organisms are recognized. d. It pertains to the use of handwashing. 44. In managing individuals with SARS, the nurse should be aware of the different signs and symptoms of the disease. Which among the following characterize the prodromal phase:
  • 16. a. The patient is usually not infectious at this time. b. Dry hacking cough is present c. Fever is present, but does not rise beyond 38oC d. The patient requires mechanical ventilation at this stage 45. Handwashing deters the spread of microorganisms. A nurse involved in the care of a patient diagnosed with SARS should: a. Perform handwashing before client contact b. Perform handwashing after eating c. Perform handwashing after food preparation d. Perform handwashing before using the toilet Situation 10: Filariasis is endemic in some parts of the Philippines. The disease often progresses to become chronic, debilitating and often unfamiliar to health workers. 46. Effective methods that the government would likely to pursue to eliminate filariasis in the country are all of the following EXCEPT: a. Pursue annual mass drug administration using two drugs in all endemic areas for at least five consecutive years b. Vaccination of all susceptible in high risk areas and high risk populations c. Intensify health information and advocacy campaigns in its prevention, control and elimination d. Halt progression of disease through disability prevention 47. The vector for Filariasis is a. Wuchereria bancrofti b. Aedes poecillus c. Anopheles d. Aedes egypti 48. A long incubation period characterizes Filariasis that typically ranges from: a. 2-4 weeks c. 2-3 years b. 4-6 weeks d. 8-16 months 49. A 36-year-old man is brought by his wife to a doctor's clinic to be tested for filariasis. The most likely diagnostic test that he will undergo is: a. Immunochromatographic test (ICT) b. Nocturnal Blood Examination c. Stool examination d. Urinalysis 50. A client in the acute stage of the disease will include which of the following clinical findings? a. Lymphangitis, lymphadenitis, epidydimitis b. Hydrocele, lymphedema, elephantiasis c. Orchitis, hydrocele, elephantiasis d. Lymphadenitis, lympedema and orchitis Situation 11: Specific defenses of the body involve the immune system. Nurses should be knowledgeable on the importance of immunity in the prevention of communicable diseases. 51. Ayisha is given the hepatitis B immune globulin serum, which will provide her with passive immunity. One advantage of passive immunity is that it: a. Has effects that last a long time b. Is highly effective in the treatment of disease c. Offers immediate protection d. Encourages the body to produce antibodies 52. Nurse Keith explains to a mother whose child just received a tetanus toxoid injection that the toxoid confers which of the following immunity? a. Lifelong passive immunity b. Long-lasting active immunity c. Lifelong active natural immunity d. Lifelong active artificial immunity 53. Simon, who is suspected of having tetanus, asks Nurse Benny about immunizations against tetanus. Nurse Benny explains that the major benefit in using tetanus antitoxin is that it:
  • 17. a. Stimulates plasma cells directly b. Provides a high titer of antibodies c. Provides immediate active immunity d. Stimulates long-lasting passive immunity 54. Justin, who was exposed to hepatitis A is given gamma globulin to provide passive immunity, which: a. Increases production of short-lived antibodies b. Provides antibodies that neutralize the antigen c. Accelerates antigen-antibody union at the hepatic sites d. Stimulates the lymphatic system to produce large numbers of antibodies 55. A mother asked Nurse Edna on the duration of the effectiveness of a natural passive immunity. Nurse Edna is correct when she tell the mother that the effect lasts for: a. 2 to 3 weeks c. 6 months to 1 year b. Permanent d. 2 to 5 years Situation 12. Whenever possible, the nurse implements strategies to prevent infection. If infection cannot be prevented, the nurse's goal is to prevent the spread of the infection within and between persons, and to treat the existing infection. 56. Gino is a chronic carrier of infection. To prevent the spread of the infection to other clients or health care providers, the nurse emphasizes interventions that do which of the following? a. Eliminate the reservoir b. Block the portal of exit from the reservoir c. Block the portal of entry into the host d. Decrease the susceptibility of the host 57. The most effective nursing action for controlling the spread of infection includes which of the following? a. Thorough hand cleansing b. Wearing gloves and masks when providing direct client care c. Implementing appropriate isolation precautions d. Administering broad-spectrum prophylactic antibiotics 58. In caring for Mark who is on contact precautions for a draining infected foot ulcer, Nurse Bong should perform which of the following? a. Wear a mask during dressing changes b. Provide disposable meal trays and silverware c. Follow standard precautions in all interactions with the client d. Use surgical aseptic technique for all direct contact with the client 59. When caring for a single client during one shift, it is appropriate for the nurse to reuse which of the following personal protective equipment? a. Goggles c. Surgical mask b. Gown d. Clean gloves 60. After teaching a client and family strategies to prevent infection, which statement by the client would indicate effective learning has occurred? a. “We will use antimicrobial soap and hot water to wash our hands at least three times per day.” b. “We must wash or peel all raw fruits and vegetables before eating.” c. “A wound or sore is not infected unless we see it draining pus.” d. “We should not share toothbrushes but it is OK to share towels and washcloths. Answer----------------------------------- 31. C. A and B. There is no known vaccination and prophylactic antivirals for Dengue fever. D. Eliminating the breeding sites of mosquitoes is better than killing the mosquitoes. 32. B 33. B. There is no known vaccination for Dengue fever. 34. A. Patient should be observed closely for signs of deterioration (shock) such as hypotension, tachycardia, tachypnea, cold clammy perspiration and prostration. B. This is not necessary. C. There is no known antivirals for Dengue fever.
  • 18. D. Platelet administration is done to prevent bleeding, but this is a dependent function. 35. B. This promotes vasoconstriction. A. Aspirin is contraindicated. It may worsen the bleeding. C. Patient should be placed in dorsal recumbent to facilitate blood circulation. D. Diet should be low fiber, low fat, non-irritating, and non-carbonated. 36. C. Tuberculin/Mantoux test only indicate exposure to the mycobacterium, not active infection. A and D. After obtaining three sputum negative results, chest x-ray can be done to assess parenchymal involvement. Diagnosis of TB through x-ray can be made by the TB Diagnostic Center (TBDC). B. Direct Sputum Smear Microscopy (DSSM) is the primary diagnostic tool for TB. 37. C. Hemoptysis is the only contraindication for sputum collection. A, B and D are indications for hospitalization of clients with TB. 38. B. The Isoniazid in the treatment regimen causes peripheral neuropathy. Client should be advised to increase intake of Vitamin B6, which is commonly found in green leafy vegetables, such as kangkong. 39. D. Yellowish palms may indicate liver problem. Anti-tuberculous drugs, particularly Pyrazinamide, are hepatotoxic drugs. A. Hemoptysis is not a side effect, but must be reported immediately to the physician. Massive hemoptysis requires hospitalization. B and C are side effects of Isoniazid. Clients should be advised to increase oral fluid intake. Paracetamol is given to manage the fever. 40. A. Strict compliance to the treatment regimen is a must while undergoing treatment for TB, to avoid relapses and prevent the development of multi-drug resistant strain of TB. B, C and D are part of the health teaching to clients with tuberculosis, but strict compliance to treatment regimen should be given utmost emphasis. 41. A. The incubation period for SARS is 2-10 days but may be long as 13 days. 42. C. A. Rhabdovirus causes rabies infection. B. Togavirus causes German measles. D. The family of paramyxovirus is responsible for measles and mumps. 43. B. A. This pertain to isolation precaution. C. This pertain to transmission-based precaution. D. Handwashing is a universal precautionary measure. 44. A. During the prodromal phase, infectivity is none to low. Infectivity is highest during the respiratory phase (within 2-7 days). B and D are seen during the respiratory stage. C. Fever during the prodromal phase of SRAS is above 380C. 45. A. Handwashing is performed before and after client contact B. Handwashing is performed before eating. C. Handwashing is performed before food preparation. D. Handwashing is performed after using the toilet. 46. B. There is no known vaccination for Filariasis. Diethycarbamezine Citrate (DEC) is given to patients with clinical manifestations and/or microfilariae. 47. B. A. This is the causative agent for filariasis. Other causative agents include Brugia malayi and Brugia timori. C. This is the vector for Malaria. D. This is the vector for Dengue. 48. D. Incubation period ranges from 8-16 months.
  • 19. 49. A. The clinic is only open from 8am to 5pm. The ICT is an antigen test that can be done in daytime. Nocturnal blood smear is also a diagnostic test for Filariasis, but the patient's blood is taken at the patient's residence or in the hospital after 8pm. 50. A. The clinic is only open from 8am to 5pm. The ICT is an antigen test that can be done in daytime. Nocturnal blood smear is also a diagnostic test for Filariasis, but the patient's blood is taken at the patient's residence or in the hospital after 8pm. 51. A. Lymphangitis, lymphadenitis, epidydimitis, funiculitis and orchotis are acute clinical manifestations of Filariasis. Hydrocele, lymphedema, and elephantiasis are clinical manifestations of Chronic Filariasis. 52. Answer: C Rationale: Passive immunity provides immediate protection, it is short-lived, is limited in effectiveness, and does not stimulate the body to produce antibodies. (NSNA NCLEX-RN Review, 4th Edition) 53. Answer: B Rationale: Tetanus antitoxin provides antibodies, which confer immediate passive artificial immunity, the same with antiserums and immune globulins. Option A is incorrect because antitoxins doesn't stimulate production of antibodies. Option C is incorrect because it provides passive, not active, immunity. Option D is incorrect because passive immunity, by definition, is not long-lasting. (Mosby's, 18th Edition) 54. Answer: B Rationale: Gamma globulin, an immune globulin, contains most of the antibodies circulating in the blood. When injected into an individual, it prevents a specific antigen from entering a host cell. Options A and D are incorrect because this does not stimulate antibody production. Option C is incorrect because this does not affect antigen-antibody function. (Mosby's, 18th Edition) 55. Answer: C Rationale: The effectiveness of a natural passive immunity lasts for 6 months to 1 year. Option B is for natural active immunity. Option A is for artificial passive immunity. Option C is for artificial active immunity. 56. Answer: B Rationale: Blocking the movement of the organism from the reservoir will succeed in preventing the infection of any other persons. In option A, since the carrier person is the reservoir and the condition is chronic, it is not possible to eliminate the reservoir. Option C and D are incorrect because blocking the entry into a host or decreasing the susceptibility of the host will be effective for only that one single individual and, thus, is not effective as blocking exit from the reservoir. 57. Answer: A Rationale: Since the hands are frequently in contact with clients and equipment, they are the most obvious source of transmission. Regular and routine hand cleansing is the most effective way to prevent movement of potentially infective materials. Option B is incorrect because personal protective equipment (PPE) such as gloves and masks is indicated for situations requiring Standard Precautions. Option C is incorrect because Isolation precautions are used for clients with known communicable diseases. Option D is incorrect because use of antibiotics is not effective and can be harmful due to the incidence of superinfection and development of resistant organisms. 58. Answer: C Rationale: Standard Precautions include all aspects of contact precautions with the exception of placing the client in a private room. In option A, a mask is indicated when working over a sterile wound rather than an infected one. Option B is incorrect because disposable food trays are not necessary for clients with infected wounds unlikely to contaminate the client's handstion D is incorrect because sterile technique (surgical asepsis) is not indicated for all contact with the client. The nurse would utilize clean technique when dressing the wound to prevent introduction of additional microbes. 59. Answer: A Rationale: Unless overly contaminated by material that has splashed in the nurse's face and cannot be effectively rinsed off, goggles may be worn repeatedly. In option B, since gowns are at high risk for contamination, they should be used only once and hen discarded or washed. Option C and D are incorrect because surgical mask and clean gloves are never washed or reused. 60.Answer: B Rationale: Raw foods touched by human hands can carry significant infectious organisms and must be washed or peeled. Option A is incorrect because antimicrobial soap is not indicated for regular use and may lead to resistant organisms. Hand cleansing should occur as needed. Hot water can dry and harm skin, increasing the risk of infection. In option C, clients should learn all the signs of inflammation and infection (e.g., redness, swelling, pain and heat) and not rely on the presence of pus to indicate this. Option D is incorrect because persons should not share washcloths or towels.